Case Study Questions Class 10 Social Science Geography Chapter 4 Agriculture

AdvantagesDisadvantages
Jhum cultivation has been practiced for generations by indigenous communities in northeastern India. It is adapted to the local subsistence farming.Jhum cultivation  often involves clearing and burning portions of forests, leading to deforestation and loss of biodiversity.
Jhum cultivation allows for the cultivation of a variety of crops in the same plot, including rice, maize, vegetables, and pulses. This diversity can help provide a more balanced diet and reduce the risk of crop failure due to pests or weather conditions.After several cycles of cultivation, the soil in jhum plots can become depleted of nutrients, leading to decreased crop yields.
Primitive subsistence farmingAgriculture is practised on small patches of land with the help of primitive tools like hoe, dao and digging sticks, and family/community labour.
Intensive subsistence farmingLabour intensive farming, where high doses of biochemical inputs and irrigation are used for obtaining higher production.
Commercial farming

 

use of higher doses of modern inputs, e.g. high yielding variety (HYV) seeds, chemical fertilisers, insecticides and pesticides in order to obtain higher productivity
RabiRabi crops are sown in winter from October to December and harvested in summer from April to June. Some of the important rabi crops are wheat, barley, peas, gram and mustard.
ZaidShort season during the summer months known as the Zaid season. Some of the crops produced during ‘zaid’ are watermelon, muskmelon, cucumber,vegetables and fodder crops.
KharifKharif crops are grown with the onset of monsoon in different parts of the country and these are harvested in September-October.

Important crops grown during this season are paddy, maize, jowar, bajra, tur (arhar), moong, urad, cotton, jute, groundnut and

Soyabean.

Case Study 5:

Leave a Reply Cancel reply

We have a strong team of experienced teachers who are here to solve all your exam preparation doubts, west bengal board class 4 maths chapter 21 solutions মাঠে টিফিন ভাগ করে খাই, west bengal board class 4 maths chapter 20 solutions নিজের খুশিমতো রং করি, jharkhand board solutions class 5 english chapter 8, jharkhand board solutions class 5 english chapter 7.

cbsencertsolutions

CBSE NCERT Solutions

NCERT and CBSE Solutions for free

Case Study Questions Chapter 4 Agriculture

Please refer to the Case Study Questions Chapter 4 Agriculture with answers provided for Class 10 Social Science. These solved case study based questions are expected to come in the Class 10 Economics exam in the current academic year. We have provided Case study for Class 10 Social Science for all chapters here. You should practise these solved case studies to get more marks in examinations.

Chapter 4 Agriculture Case Study Questions Class 10 Social Science

1. Read the source given below and answer the following questions:

Globalisation is not a new phenomenon. It was there at the time of colonisation. In the nineteenth century when European traders came to India, at that time too, Indian spices were exported to different countries of the world and farmers of south India were encouraged to grow these crops. Till today it is one of the important items of export from India. During the British period cotton belts of India attracted the British and ultimately cotton was exported to Britain as a raw material for their textile industries. Cotton textile industry in Manchester and Liverpool flourished due to the availability of good quality cotton from India. You have read about the Champaran movement which started in 1917 in Bihar. This was started because farmers of that region were forced to grow indigo on their land because it was necessary for the textile industries which were located in Britain. They were unable to grow foodgrains to sustain their families. Under globalisation, particularly after 1990, the farmers in India have been exposed to new challenges. Despite being an important producer of rice, cotton, rubber, tea, coffee, jute and spices our agricultural products are not able to compete with the developed countries because of the highly subsidised agriculture in those countries. Today, Indian agriculture finds itself at the crossroads. To make agriculture successful and profitable, proper thrust should be given to the improvement of the condition of marginal and small farmers. The green revolution promised much. But today it’s under controversies. It is being alleged that it has caused land degradation due to overuse of chemicals, drying aquifers and vanishing biodiversity. The keyword today is “gene revolution”, which includes genetic engineering.

Answer the following MCQs by choosing the most appropriate option.

(i) What do you understand by ‘gene revolution’? (a) white revolution (b) green revolution (c) genetic engineering (d) all the above

(ii) What was grown in Champaran? (a) Indigo (b) Cotton (c) Jute (d) all the above

(iii) In the 19th century which concept was not new? (a) liberalisation (b) Democracy (c) Globalisation (d) None of the above

(iv) Cotton textile industry in ______________ and Liverpool flourished due to the availability of good quality cotton from India. (a) Boston (b) Manchester (c) California (d) New York

2. Read the source given below and answer the questions that follows:

The main characteristic of this type of farming is the use of higher doses of modern inputs, e.g. high yielding variety (HYV) seeds, chemical fertilisers, insecticides and pesticides in order to obtain higher productivity. The degree of commercialisation of agriculture varies from one region to another. For example, rice is a commercial crop in Haryana and Punjab, but in Odisha, it is a subsistence crop. Plantation is also a type of commercial farming. In this type of farming, a single crop is grown on a large area. The plantation has an interface of agriculture and industry. Plantations cover large tracts of land, using capital intensive inputs, with the help of migrant labourers. All the produce is used as raw material in respective industries. In India, tea, coffee, rubber, sugarcane, banana, etc., are important plantation crops. Tea in Assam and North Bengal coffee in Karnataka are some of the important plantation crops grown in these states. Since the production is mainly for market, a welldeveloped network of transport and communication connecting the plantation areas, processing industries and markets plays an important role in the development of plantations.

(i) Which one of the following is the example of plantation agriculture? (a) Jute (b) Wheat (c) Tea (d) Oilseeds

(ii) Which of the following describes a system of agriculture, where a single crop is grown on a large area? (a) Shifting agriculture (b) Plantation agriculture (c) Horticulture (d) Intensive agriculture

(iii) Which of the following crops is the main source of jaggery, khandsari and molasses? (a) Arhar (b) Coconut (c) Linseed (d) Sugarcane

(iv) Which one of the following crops is a beverage crop? (a) Tea (b) Cotton (c) Wheat (d) Bajra

3. Read the source given below and answer the questions that follows:

You have studied the physical diversities and plurality of cultures in India. These are also reflected in agricultural practices and cropping patterns in the country. Various types of food and fibre crops, vegetables and fruits, spices and condiments, etc. constitute some of the important crops grown in the country. India has three cropping seasons — rabi, kharif and zaid. Rabi crops are sown in winter from October to December and harvested in summer from April to June. Some of the important rabi crops are wheat, barley, peas, gram and mustard. Though, these crops are grown in large parts of India, states from the north and north-western parts such as Punjab, Haryana, Himachal Pradesh, Jammu and Kashmir, Uttarakhand and Uttar Pradesh are important for the production of wheat and other rabi crops. Availability of precipitation during winter months due to the western temperate cyclones helps in the success of these crops. However, the success of the green revolution in Punjab, Haryana, western Uttar Pradesh and parts of Rajasthan has also been an important factor in the growth of the above mentioned rabi crops. Kharif crops are grown with the onset of monsoon in different parts of the country and these are harvested in September-October. Important crops grown during this season are paddy, maize, jowar, bajra, tur (arhar), moong, urad, cotton, jute, groundnut and soyabean. Some of the most important ricegrowing regions are Assam, West Bengal, coastal regions of Odisha, Andhra Pradesh, Telangana, Tamil Nadu, Kerala and Maharashtra, particularly the (Konkan coast) along with Uttar Pradesh and Bihar. Recently, paddy has also become an important crop of Punjab and Haryana. In states like Assam, West Bengal and Odisha, three crops of paddy are grown in between the rabi and the kharif seasons, there is a short season during the summer months known as the Zaid season. Some of the crops produced during ‘zaid’ are watermelon, muskmelon, cucumber, vegetables and fodder crops. Sugarcane takes almost a year to grow.

(i) Rabi crops are: (a) sown in winter and harvested in summer (b) sown during rainy season and harvested in winter (c) sown in summer and harvested in winter (d) None of the above

(ii) Match the following

1. Kharif(a) Watermelon
2. Rabi (b) Aus
3. Zaid (c) millet
4. paddy crop(d) oilseeds

Choose the correct option: (a) 1–(d), 2–(c), 3–(b), 4–(c) (b) 1–(b), 2–(c), 3–(d), 4–(a) (c) 1–(c), 2–(d), 3–(a), 4–(b) (d) 1–(c), 2–(a), 3–(b), 4–(d)

(iii) Which of the following crops is produced during zaid cropping season? (a) sugarcane (b) muskmelon (c) groundnut (d) moong

(iv) Which of the following is a kharif crop? (a) Barley (b) Peas (c) Bajra (d) Mustard

4. Identify the soil with the help of the following features. * also known as regur soil. * made of extremely fine i.e. clayey material * ideal for growing cotton

5. Read the source given below and answer the following questions:

Agriculture has been practised in India for thousands of years. Sustained uses of land without compatible techno-institutional changes have hindered the pace of agricultural development. Inspite of development of sources of irrigation most of the farmers in large parts of the country still depend upon monsoon and natural fertility in order to carry on their agriculture. For a growing population, this poses a serious challenge. Agriculture which provides livelihood for more than 60 per cent of its population, needs some serious technical and institutional reforms. Thus, collectivisation, consolidation of holdings, cooperation and abolition of zamindari, etc. were given priority to bring about institutional reforms in the country after Independence. ‘Land reform’ was the main focus of our First Five Year Plan. The ‘right of inheritance’ had already led to fragmentation of land holdings necessitating consolidation of holdings.

(i) What do you understand by the term ‘collectivisation’? (a) Combining land of various farmers and then performing the agricultural activities on collective basis (b) Combining land of various farmers and then performing the agricultural activities on individual basis (c) Comparing land of various farmers and then performing the agricultural activities on pocket basis (d) none of the above

(ii) Which of the following is the most important occupation of the people of India? (a) Food gathering (b) Agriculture (c) Manufacturing (d) Services

(iii) What was the main objective of First Five Year Plan? (a) Land forms (b) Land degradation (c) Land reforms (d) all the above

(iv) In spite of development of irrigation , farmers still depend on ____________. (a) artificial water resources (b) monsoon (c) wells (d) none of the above

Very Short Answer Type Questions

Question. Which crop is known as ‘golden fibre’? Answer :  Jute

Question. Complete the following table with correct information with regard to cultivation of Rice:

Chapter 4 Agriculture

Answer :  (A) Kharif Cropping Season (B) 16°C – 27°C.

Question. Write the amount of annual rainfall required for the cultivation of wheat. Answer :  50-75 cm of rainfall is the amount of rainfall required for the cultivation of wheat.

Question. Write the temperature requirement of the maize crop. Answer :  21-27 degrees Celsius is the temperature requirement of the maize crop.

Short Answer Type Questions

Question. What are the main features of plantation agriculture? Explain. Answer :  Main features of plantation agriculture are as follows: (1) Plantation is a type of commercial farming in which a single crop is grown on a large area. (2) Plantations cover large tracts of land using capital intensive inputs with the help of migrant labourers. (3) All the produce in this farming is used as raw material in respective industries. Cash crops like tea, coffee, rubber, sugarcane, banana, spices etc. are produced on large scale of sale in the market for which it requires a welldeveloped system of transport and communication.

Question. Describe geographical conditions required for tea cultivation. Answer :  Tea Cultivation: (1) Tea grows well in tropical and subtropical climates. (2) Fertile well drained soil. (3) Warm and moist frost-free climate. (4) Frequent showers throughout the year. (5) Intensive labour.

Question. Highlight any three differences between primitive subsistence farming and commercial farming. Answer : A. Commercial Farming (1) Use of higher doses of modern inputs like HYV seeds, fertilizers, etc. (2) Commercialization of agriculture varies from region to another (3) Plantation is also a type of commercial farming (4) Use of well-developed network of transport and communication (5) High productivity for commercial purpose B. Subsistence Farming (1) Is practiced on small patches of land (2) Labour intensive farming (3) Use of primitive tools (4) Dependent on Monsoons (5) Called as ‘Slash and burn’ agriculture. (6) Low productivity

Question. Wheat and rice farming in India are fairly different from each other. Support the statement.

Chapter 4 Agriculture

Answer :  The differences between Wheat and Rice Cultivation are explained in the table:

Question. Which crop in India is known as golden fibre? Mention any two geographical conditions that it requires for its growth. What are its uses? Answer :  Jute is called the golden fibre because of its high cash value in the market. Geographical conditions for its growth are as follows: (1) Jutes grows well in well-drained fertile soils in the flood plains where soils are renewed every year. (2) High temperature is required during the time of growth Uses of Jute are as follows: (3) Jute is used in making gunny bags, mats, ropes, yarn, carpets and other artifacts.

Question. Describe the technological reforms taken by the Indian Government in the field of agriculture. Answer :  To improve Indian agriculture, the government of India began introducing agricultural and institutional reforms in the 1960s. Some of them have been listed below – (1) Widespread use of radio and television for providing knowledge to farmers bout new and improved techniques of cultivation and introduction of special weather bulletins. (2) Crop insurance to protect the farmers’ prouduce against losses by natural and man-made calamities like fire, flood, drought, etc. was introduced. (3) To ensure easy availability of capital or investment, the Grameen Bank and cooperative societies were established. They provide credit to farmers at very low rates of interest. (4) Kisan Credit Card (KCC) and Personal Accident Insurance Scheme (PAIS) have been introduced by the government for the benefit of farmers. (5) Announcement of procurement or remunerative prices, minimum support prices for farmers to reduce exploitation of farmers by middlemen.

Question. Read the sources given below and answer the questions that follows: Source A: Types Of Farming This type of farming is still practiced in few pockets of India. It is practiced on small patches of land with the help of primitive tools like hoe, dao and digging sticks, and family community labour. This type of farming depends upon monsoon, natural fertility of the soil and suitability of other environmental conditions to the crops grown. Source B: Commercial Farming In India, tea, coffee, rubber, sugarcane, banana, etc., are important plantation crops. Tea in Assam and North Bengal coffee in Karnataka are some of the important plantation crops grown in these states. Since the production is mainly for market, a well- developed network of transport and communication connecting the plantation areas, processing industries and markets plays an important role in the development of plantations. Source C: Cropping Pattern In states like Assam, West Bengal and Odisha, three crops of paddy are grown in a year. These are Aus, Aman and Boro. In between the rabi and the kharif seasons, there is a short season during the summer months known as the Zaid season. Some of the crops produced during ‘zaid’are watermelon and muskmelon. Source A: Types of Farming (A) Identify the tyoe of Farming through its description in the source? Source B: Commercial Farming (B) What do you mean by Plantation Crops? Source C: Cropping Pattern (C) Name One Zaid Crop? Answer :  (A) This type of farming is Primitive Subsistence Farming. This type of farming is only used for consumption and subsistence purposes. (B) A plantation crop is a crop which is cultivated on large scale farms called plantations for export purposes. Plantation farming is a kind of commercial farming and is mainly done for trade. Coffee is a plantation crop. (C) Another example of Zaid Crop is Cucumber.

Question. State any three characteristics of commercial farming in India. Answer :  The main characteristics of Commercial farming in India are as follows: (1) Crops are grown on a large scale in large farms or plantations for commercial purposes and for export to other countries. (2) It is a capital intensive farming and requires high application of modern inputs. (3) Modern inputs like high yielding variety (HYV)seeds, chemical fertilisers, insecticides and pesticides are used to obtain higher productivity. Irrigation is also used to meet water requirements.

Question. Categorise the following as kharif crops and rabi crops: (A) Wheat (B) Maize (C) Barley (D) Peas (E) Bajra (F) Tur (arhar) Answer :  (A) Wheat – Rabi crop (B) Maize – Kharif crop (C) Barley – Rabi crop (D) Peas – Rabi crop (E) Bajra – Kharif crop (F) Tur (arhar) – Kharif crop

Question. ‘India is an agriculturally important country.’ Justify the statement with three arguments. Answer :  Arguments are as follows: (1) Two-thirds of its population is engaged in agricultural activities. (2) Agriculture is a primary activity which produces most of the food that we consume. (3) Besides food grains, it also produces raw material for various industries. (4) Some agricultural products like tea, coffee, spices etc. are also exported which brings much needed foreign exchange in the country.

Long Answer Type Questions

Question. What steps have been taken by Indian government to modernise agriculture? Explain. Answer :  India is essentially an agricultural country where two-third of its total population are engaged in agricultural activities. Considering, the importance of agriculture in India, following steps have been taken by the government to modernise it: (1) Governments has established Indian Council of Agricultural Research. (2) It has established agricultural universities, veterinary services and animal breeding centres. (3) It has given priority to research and development in the field of meteorology and weather forecast. (4) Rural infrastructure has been improved. (5) Cheap credit facility has been provided to Indian farmers to buy necessary inputs like machines, fertilizers, seeds etc.

Question. Why are maximum Jute textile mills located in the Hugli Basin? Answer :  Location of Jute industries in Hugli basin: (i) Proximity of jute producing areas. (ii) Inexpensive water transport. (iii) Good network of roadways, railways and waterways (iv) Abundant water for processing raw jute (v) Cheap labour from West Bengal and adjoining states (vi) Good banking, insurance and poor facilities in Kolkata.

Question. Compare ‘intensive subsistence farming’ with that of ‘commercial farming’ practiced in India. Answer :  Comparison between ‘Intensive Subsistence farming’ and ‘Commercial farming’: (1) In Intensive subsistence farming pressure of population on land is high whereas in commercial farming population pressure is low. (2) In intensive subsistence farming labour intensive farming is used whereas in commercial farming mechanized form of farming is used. (3) In intensive subsistence farming there is low capital investment whereas in commercial farming high capital investment is seen. (4) In intensive subsistence farming farmers produce for their own consumption whereas in commercial farming production is mainly for the market. (5) In intensive subsistence farming processing industries are not associated with farms whereas in commercial farming processing industries are associated with plantations. (6) In intensive subsistence farming multiple cropping is practiced whereas in commercial farming single cropping is practiced. (7) In intensive subsistence farming land holdings are small whereas in commercial farming land holdings are large.

Question. Describe any five features of primitive subsistence farming. Answer :  The features of primitive subsistence farming are: (1) Primitive subsistence farming is a type of farming practice in which the farmer and his family raise crops for home consumption and not trade. (2) This is practiced with the help of primitive tools like hoe, dao and digging sticks. The farming totally depends on the monsoon and natural fertility of the soil. (3) It is done on small areas of land and is labour intensive. (4) It is practiced as a ‘slash and burn’ agriculture. Farmers clear a patch of land and produce crops for their sustenance. (5) Land productivity is low. No artificial fertilizers are used.

Question. Read the extract given below: The main characteristic of this type of farming is the use of higher doses of modern inputs, e.g. high yielding variety (HYV) seeds, chemical fertilisers, insecticides and pesticides in order to obtain higher productivity. The degree of commercialisation of agriculture varies from one region to another. For example, rice is a commercial crop in Haryana and Punjab, but in Odisha, it is a subsistence crop. Plantation is type of commercial farming. In this type of farming, a single crop is grown on a large area. The plantation has an interface of agriculture and industry. Based on the above comprehension, answer the following questions: (A) Identify the kind of Farming mentioned in the passage. (B) What do you mean by a Subsistence Crop? (C) How can you say that Plantation has an interface of both agriculture and industry? Answer :  (A) The kind of Farming mentioned in the passage is Commercial Farming. (B) Crop grown for consumption and not for sale purpose is called Subsistence Crop. All food crops are Subsistence crops except if they are grown in surplus. For example: Wheat, Rice etc. (C) Plantation is an interface of industry and agriculture because in this type of farming, crops are grown using advanced agricultural practices for the purpose of export and trade. The products of Plantation farming are mostly used for selling and not for consumption and hence support various food processing industries. It also provides raw materials to a number of agro industries. This is why Plantation farming is both agriculture and industry mixed into one.

Question. Name the two major fibre crops grown in India. Describe the conditions required for the growth of these two crops with their growing areas. Answer :  Major fibre crops grown in India are: (1) Cotton (2) Jute (3) Hemp (4) Natural silk

The conditions required for the growth of the following crops are:

(1) Cotton: (i) Cotton grows well in drier parts of the black cotton soil of the Deccan plateau. It requires high temperature, light rainfall or irrigation. (ii) It needs 210 frost-free days and bright sunshine for its growth. The leading cotton producing states are Gujarat, Maharashtra, Andhra Pradesh, Punjab, Haryana, Karnataka, Tamil  Nadu and Madhya Pradesh.

(2) Jute: (i) Jute grows well on well-drained fertile soil in flood plains, where soil is renewed every year. (ii) High temperature is required during the time of growth. Jute is grown in West Bengal, Bihar, Assam, Odisha and Meghalaya.

Question. “The Government of India has introduced various institutional and technological reforms to improve agriculture in the 1980s and 1990s.” Support this statement with examples. Answer :  Reforms made in agriculture sectors are as follows: (1) Land reforms such as collectivisation, consolidation of land holdings, cooperation and abolition of zamindari system has been given priority. (2) Provision for crop insurance against drought, flood, cyclone, fire and disease. (3) Established Grameen banks, cooperative societies and banks for providing loan facilities to the farmers at lower rates of interest. (4) Government has introduced Kissan Credit Card (KCC), Personal Accident Insurance Scheme (PAIS) for the benefit of the farmers in our country. (5) Special Weather Bulletins and agricultural programmes such as Krishi Darshan for farmers were introduced on the radio and television. (6) The government also announces minimum support price, remunerative and procurement prices for importan crops to check the exploitation of farmers by speculators and middlemen.

Question. What are the main characteristic features of intensive subsistence farming? Mention any five of them. Answer :  Main characteristics features of Intensive Subsistence Farming: (1) This type of farming is practiced in areas of high population pressure on land. (2) It is labour intensive farming (3) High doses of biochemical inputs and irrigation are used for obtaining higher production. (4) Due to the law of inheritance division of land among successive generations has rendered land holding size uneconomical (5) There is enormous pressure on agriculture land because the farmers continue to take maximum output from the limited land in the absence of alternative source of livelihood.

Question. Name the two major beverage crops grown in India. Answer :  Tea and coffee are the two major beverage crops grown in India. Tea and coffee are grown in: (1)  Coffee:  Coffee grows in areas having with temperature between 15°C and 28°C and rainfall of about 150 cm to 200 cm. The area should have rich, well drained and loamy soil. In India, it grows in hills around Nilgiris, Karnataka, Kerala and Tamil Nadu. (2)  Tea:  Tea grows in areas with temperature between 20°C to 30°C and rainfall of about 150 cm to 200 cm. It grows in welldrained soil. In India, it grows in Assam, West Bengal, Tamil Nadu and Kerala.

Case Study Questions Chapter 4 Agriculture

Related Posts

Acids Bases Salts Class 10 Science Important Questions

Acids Bases Salts Class 10 Science Important Questions

English Class 10

Important Questions Class 10 English with Answers

Case Study Questions Chapter 2 Federalism

Case Study Questions Chapter 2 Federalism

Rankers Study Tutorial

What is Case Study Question / Paragraph Based Question? A case study is a scenario in a particular academic / professional context which students are expected to analyse and respond to, guided by specific questions posed concerning the situation. In many cases, the scenario or case study involves a number of issues or problems that must be dealt with in a academic / professional workplace.

Why Case Study Questions are included in academics? Case study assignments usually require students to identify problems and issues in a scenario, to demonstrate their developing knowledge of theories and academic / professional policies and to make decisions and recommendations based on these to either prevent or solve some of the issues in that scenario.

How to solve Case Study Questions? There are several steps to writing an answer to a case study assignment:

STEP 1: Read the case study and questions carefully.

  • Read the case and associated questions carefully.
  • Highlight the main points of the case and any issues that you can identify.
  • Read the questions closely and analyse what they are requiring you to do.
  • Read the case again, linking the information that is relevant to each question you have been asked.

STEP 2: Identify the issues in the case study. Case studies describe a situation which may arise in a particular profession or social context. They often involve a number of people in a complex situation. They will often describe a situation which is problematic, possibly in how it is dealt with, or in its complexity. An important part of your answer is to analyse the situation and to identify the issues/actions described in the case which may be problematic. The following questions may help you to do this:

  • What actions were taken in the case?
  • Were these actions the most appropriate and why?
  • Were there any consequences of the actions taken?
  • Was anything omitted or not considered?
  • Were actions/procedures in line with existing codes of practice, policy or theories?

STEP 3: Link theory to practice. Use your knowledge of existing codes of practice, theories and/or other academic / professional documents and behaviours to decide what was done appropriately and what was not.

STEP 4: Plan your answer. It can be useful to use the questions you have been set as headings and to answer each part in turn, reducing the chance of omitting set questions. You can always take out the headings before you submit if you wish. Lecturers usually set questions in a logical order, so answer in the order they are written in your question.

STEP 5: Start writing your case study answer (for theory only) Like any assignment, you will need an introduction, body sections in which you answer the questions put to you regarding the case study, and a conclusion.

STEP 6: Edit and proofread. Read through your paper yourself to detect and correct other errors and omissions. Check you have answered all questions and backed up your answer with relevant passage.

Types of Case Study Questions / Paragraph Based Questions Case Study Questions / Paragraph Based Questions can be broadly classified into two types:

  • MCQs type: In this type, student has to tick the correct option from various options.
  • Theory type: In this type, student has to write proper solution / answer in cotext to the case study.

Case Study/ Passage Based Questions Chapter 4 Agriculture

Type 1: MCQ type

Case Study Question 01

Read the extract given below and answer the questions that follow:

Primitive Subsistence Farming : This type of farming is still practised in a few pockets of India. Primitive subsistence agriculture is practised on small patches of land with the help of primitive tools like a hoe and digging sticks, and family/community labour. This type of farming depends upon monsoon, natural fertility of the soil and suitability of other environmental conditions to the crops grown. Intensive Subsistence Farming: This type of farming is practised in areas of high population pressure on land. It is labour intensive farming, where high doses of biochemical inputs and irrigation are used for obtaining higher production. Though the ‘right of inheritance’ leading to the division of land among successive generations has rendered land-holding size uneconomical, the farmers continue to take the maximum output from the limited landin the absence of an alternative source of livelihood. Thus, there is enormous pressure on agricultural land. Commercial Farming: The main characteristic of this type of farming is the use of higher doses of modern inputs, e.g. high yielding variety (HYV) seeds, chemical fertilisers, insecticides and pesticides in order to obtain higher productivity. The degree of commercialisation of agriculture varies from one region to another. For example, rice is a commercial crop in Haryana and Punjab, but in Odisha, it is a subsistence crop. Plantation: Plantation is also a type of commercial farming. In this type of farming, a single crop is grown on a large area. The plantation has an interface of agriculture and industry. Plantations cover large tracts of land, using capital intensive inputs, with the help of migrant labourers. All the produce is used as a raw material in the respective industries. Tea in Assam and North Bengal coffee in Karnataka are some of the important plantation crops grown in these states.

Answer the following MCQs by choosing the most appropriate option:

Question.1. Tea in Assam: Plantation Crop, Rice in Odisha: __________.

(a) Primitive Subsistence crop (b) Commercial crop (c) Plantation crop (d) Subsistence crop

Question.2. Which of the following is not correct about plantation farming?

(a) In this type of farming, a single crop is grown on a large area. (b) The plantation has an interface of agriculture and industry. (c) Plantations cover large tracts of land called estates. (d) Farmers clear a patch of land by felling trees and burning them, to produce cereals and other food crops.

Question.3. ________ is done in a mechanised way using capital intensive inputs and particularly timely irrigation making maximum use of large tracts of land.

(a) Intensive Subsistence Farming (b) Plantation Farming (c) Commercial Farming (d) Jhum Farming

(a) Farmers use chemical fertilisers, insecticides and pesticides. (b) Farmers use high doses of biochemical inputs. (c) Farmers use simple and basic tools and irrigation methods. (d) Farmers use mechanised tools and machines.

Ans.1. (d) Subsistence crop Ans.2. (d) Farmers clear a patch of land by felling trees and burning them, to produce cereals and other food crops. Ans.3. (b) Plantation Farming Ans.4. (c) Farmers use simple and basic tools and irrigation methods.

Case Study Question 02

Rabi crops are sown in winter from October to December and harvested in summer from April to June. Some of the important rabi crops are wheat, barley, peas, gram and mustard. Though, these crops are grown in large parts of India, states from the north and north-western parts such as Punjab, Haryana, Himachal Pradesh, Jammu and Kashmir, Uttarakhand and Uttar Pradesh are important for the production of wheat and other rabi crops. Availability of precipitation during winter months due to the western temperate cyclones helps in the success of these crops. However, the success of the green revolution in Punjab, Haryana, western Uttar Pradesh and parts of Rajasthan has also been an important factor in the growth of the above mentioned rabi crops.

Answer the following MCQs by choosing the most appropriate option :

Question.1. Rabi crops are sown in _______ and harvested in ________.

(a) A-(1), B-(2), C-(3), D-(4) (b) A-(4), B-(3), C-(1), D-(2) (c) A-(2), B-(3), C-(4), D-(1) (d) A-(3), B-(4), C-(1), D-(2)

Question.3. Which type of impact does summer rainfall put on Rabi crops?

(a) Positive (b) Negative (c) No Effect (d) None of these

Question.4. Which of the following factor has given boost to Rabi crops in Northwest India?

(a) Demand by Population (b) Awareness of Farmers (c) Rainfall (d) Green Revolution

Ans.1. Winter, Summer Ans.2. (c) A-(2), B-(3), C-(4), D-(1) Ans.3. (c) No Effect Ans.4. (d) Green Revolution

Case Study Question 03

We understand the physical diversities and plurality of cultures in India. These are also reflected in agricultural practices and cropping patterns in the country. Various types of food and fibre crops, vegetables and fruits, spices and condiments, etc. constitute some of the important crops grown in the country. India has three cropping seasons — rabi, kharif and zaid.

A variety of food and non-food crops are grown in different parts of the country depending upon the variations in soil, climate and cultivation practices. Major crops grown in India are rice, wheat, millets, pulses, tea, coffee, sugarcane, oilseeds, cotton and jute, etc.

Question.1. Millets come in the category of ________ crops.

(a) food (b) non-food (c) bevarage (d) None of these

Choose the correct option:

(a) a-2, b-1, c-3 (b) a-3, b-2, c-1 (c) a-1, b-3, c-2 (d) a-3, b-1, c-2

Question.3. Which one of the following is not a physical feature that influence cropping pattern?

(a) Soil (b) Terrain (c) Dense population (d) Air moisture

Question.4. In order to increase the crop output, a farmer can control _________.

(a) Cultivation practices (b) Sunlight (c) Humidity (d) Physiography

Ans.1. (a) food Ans.2. (b) a-3, b-2, c-1 Ans.3. (c) Dense population Ans.4. (a) Cultivation practices.

Type 2: Theory Type

Case Study Question 04

Read the extract and answer the questions that follows:

Intensive Subsistence Farming is practised in areas of high population pressure on land. It is labour intensive farming, wherehigh doses of biochemical inputs and irrigation are used for obtaining higher production. Though the ‘right of inheritance’ leading to the division of land among successive generations has rendered land holding size uneconomical, the farmers continue to take maximum output from the limited land in the absence of alternative source of livelihood. Thus, there is enormous pressure on agricultural land. The main characteristics of commercial farming is the use of higher doses of modern inputs, e.g. high yielding variety (HYV) seeds, chemical fertilisers, insecticides and pesticides in order to obtain higher productivity. The degree of commercialisation of agriculture varies from one region to another. For example, rice is a commercial crop in Haryana and Punjab, but in Odisha, it is a subsistence crop.

Question.1. What leads to the division of land among successive generations has rendered land holding size uneconomical?

Question.2. How does commercial farming maximise it productivity?

Ans.1. The ‘Right of inheritance ‘leads to the division of land among successive generations has rendered land-holding size uneconomical. Ans.2. (i) The main characteristics of commercial farming is the use of higher doses of modern inputs. (ii) Some of the examples are high yielding variety (HYV) seeds, chemical fertilisers, insecticides and pesticides in order to obtain higher productivity.

Related Posts

Competency based questions chapter 5 body movements, revision notes for class 10 geography – chapter 3 water resources, competency based questions chapter 11 electricity, leave a reply cancel reply.

Your email address will not be published. Required fields are marked *

Save my name, email, and website in this browser for the next time I comment.

You cannot copy content of this page

Javascript not detected. Javascript required for this site to function. Please enable it in your browser settings and refresh this page.

NCERT Solutions for Class 6, 7, 8, 9, 10, 11 and 12

Social Science Class 10 Important Questions Geography Chapter 4 Agriculture

October 1, 2019 by Sastry CBSE

Question 1. Write four points to stress the importance of agriculture in India. (2013) Or, Why is India called an agriculturally important country? Write four supportive arguments. Or, ‘Agriculture has been the backbone of the Indian Economy’ Explain the statement by giving reason. (2017 D) Answer: India is an agricultural country because of the following reasons:

  • Two-third of its population is engaged in agricultural activities which provide livelihood.
  • Agriculture is a primary activity and produces most of the food and foodgrains that we consume.
  • It produces raw materials for our various industries, e.g., cotton textile and sugar industry.
  • Some agricultural products, like tea, coffee and spices, are exported and earn foreign exchange.
  • The share of agriculture in providing employment and livelihood to the population continued to be as high as 63% in 2001.

Question 2. Why do farming practices vary in different regions? Give three major reasons. Answer: Three major reasons are:

  • Physical environment, i.e., relief, soil and climate.
  • Technological know-how.
  • Socio-cultural practices.

Question 3. Name some industries based on agricultural raw material. Answer: Cotton Textile Industry and Sugar Industry.

Question 4. What factors does primitive subsistence farming depend on? Answer: Factors:

  • Natural fertility of the soil
  • Suitability of other environmental conditions to the crops grown.

Question 5. Write main characteristics of ‘slash and burn’ agriculture. Or, Write main characteristics of Primitive Subsistence Farming. Answer:

  • This type of farming is practised in few pockets of India on small patches of land using primitive tools and family/community labour.
  • Farmers clear a patch of land and produce cereals and other food crops to sustain their families.
  • When the soil fertility decreases, the farmers shift and clear a fresh patch of land for cultivation. This type of shifting allows nature to replenish the fertility of the soil through natural processes.
  • Land productivity is low as the farmer does not use fertilisers or other modem inputs.

Question 6. Name some primitive tools used in ‘slash and bum’ agriculture. Answer: Tools like hoe, dao and digging sticks.

Question 7. Compare ‘intensive subsistence farming’ with that of ‘commercial farming’ practiced in India. Answer:

Intensive Subsistence Farming Commercial Farming
(i) In this type of farming, production is mainly for self consumption. (i) In this, crops are mainly grown for commercial purposes.
(ii) It is practised in areas of high population pressure on land. (ii) It is practiced on large pieces of land on scientific and commercial lines.
(iii) It is labour intensive farming. (iii) In this type of agriculture, machines and modem technology are used.
(iv) In this, high doses of biochemical inputs and irrigation are used for obtaining higher production. (iv) There is higher use of modern agricultural inputs, for example, High Yielding Variety (HYV) seeds, fertilisers, irrigation, etc. are used to obtain higher yields and production.
(v) Farmers take maximum output from the limited land by raising 2-3 crops in a year from the same land, i.e., multiple cropping is practised. (v) The degree of commercialization varies from one region to another. Rice is a commercial crop in Punjab, while in Orissa it is subsistence crop. For example, Plantation agriculture.

Question 8. Write the main characteristics of intensive subsistence farming. Answer:

  • It is practised in areas of high population pressure on land.
  • It is labour intensive farming.
  • Yield per hectare is high because high doses of biochemical inputs and irrigation are used.
  • The size of the land-holdings is small and uneconomical.
  • Farmers take maximum output from the limited land by raising 2-3 crops in a year from the same land, i.e., multiple cropping is practised.

Question 9. What are the major differences between primitive subsistence farming and commercial farming? (2013) Answer:

Primitive Subsistence Farming Commercial Farming
1. In this mainly cereals and other food crops are grown by farmers to sustain themselves. 1. In this, crops are mainly grown for commercial purposes.
2. It is generally done on small land holdings which are economically not viable. 2. It is practiced on large pieces of land on scientific and commercial lines.
3. Primitive tools and animals are used for carrying out agricultural activities. 3. In this type of agriculture, machines and modern technology are used.
4. In this, modern agricultural inputs, e.g., fertilisers and irrigation are not widely used. 4. There is higher use of modern agricultural in­puts, e.g., HYV seeds, fertilisers, irrigation, etc. are used to obtain higher yields and production.
5. Subsistence Agriculture is called Jhumming in Assam, Mizoram; Kuruwa in Jharkhand.

E.g., Slash and Burn.

5. The degree of commercialization varies from one region to another. Rice is a commercial crop in Punjab, while in Orissa it is subsistence crop. E.g., Plantation agriculture.

Question 10. What is the most important characteristic of commercial farming? Answer: The main characteristic of commercial farming is the use of higher doses of modern inputs, e.g., high yielding varieties seeds (HYVs), chemical fertilizers and pesticides to increase productivity. The degree of commercialisation of agriculture varies from one region to another.

Question 11. Give an example of a crop which is commercial in one region and provides subsistence in another. (2012) Answer: Rice is a commercial crop in Haryana and Punjab, while in Odisha it is a subsistence crop.

Question 12. Name one type of agriculture which falls in the category of commercial agriculture. Write the main characteristics of this type of agriculture. (2012) Or Write six characteristics of plantation agriculture. Answer: Plantation agriculture is a type of commercial farming. Characteristics of plantation agriculture:

  • A single type of crop is grown on a large area.
  • Plantation is carried out on large estates using lot of capital intensive units.
  • Lot of migrant labourers work on these estates.
  • The plantation has an interface of agriculture and industry. All the produce is used as raw material in the respective industries.
  • The production is mainly for the market, i.e., commercial agriculture.
  • A well developed network of transport and communication connecting the plantation areas, processing industries and markets plays an important role in the development of plantations.

Question 13. Name one horticultural plantation crop and two beverage plantation crops and two States each which specialise in their production respectively. Answer: Horticultural plantation crop is apples, mainly grown in Himachal Pradesh and Jammu & Kashmir. Beverage crops:

  • Tea which is mainly a plantation crop in Assam and North-West Bengal.
  • Coffee which is grown in Karnataka.

Question 14. Name some important plantation crops. Answer: Tea, coffee, rubber, sugarcane and banana are examples of some important plantation crops.

Question 15. Name three cropping seasons of India. Write their sowing and harvesting time and major crops of each season. Answer: Rabi, Kharif and Zaid are the three cropping seasons.

I. Rabi crops (Winter crops): These are sown in winter from October to December. Harvested in summer from April to June. Important crops are wheat, barley, mustard, peas, gram etc.

II. Kharif crops (Crops of the rainy season): These are sown with the onset of monsoon (June-July) and are harvested in September-October. Important crops are rice, maize, millets, jowar, bajra, tur (arhar), moong, urad, cotton, jute, groundnut and soyabean.

III. Zaid season: It is a short cropping season during summer months (mainly between March-April and June-July in different parts of the country). Important crops are watermelon, musk melon, cucumber and fodder crops, etc.

Question 16. Where are rabi crops mainly grown? Describe the climatic conditions required for their growth. Answer: Rabi crops such as wheat and other crops are grown mainly in states from the north and north-western parts such as Punjab, Haryana, Himachal Pradesh, Jammu & Kashmir, Uttaranchal and Uttar Pradesh. Climatic conditions: Availability of precipitation during winter months due to western temperate cyclones help in successful growth. The green revolution has also been an important factor in the growth of rabi crops.

Question 17. Name some important rice growing regions of India. Answer: Assam, West Bengal, Coastal regions of Orissa, Andhra Pradesh, Tamil Nadu, Kerala and Maharashtra etc.

Question 18. Name three states which raise three paddy crops in a year. Name these crops also. Answer: Assam, West Bengal and Orissa grow three crops in a year. The crops are Aus, Aman and Boro.

Question 19. (a) Which is the most important food crop of India? What is its position in world production? Or, Name the food crop of Kharif season. What is India’s position in the world with regard to its production? (b) Write the rainfall and temperature requirements for the growth of this crop. (c) Name four major regions of rice cultivation. (d) Name four states which produce irrigated rice. Answer: (a) Rice is the most important food crop (Kharif crop) of India. India holds second position in rice production after China.

(b) For rice cultivation, high temperature of 25°C and above and high humidity with annual rainfall of 100 cms is required.

(c) Four major regions of rice cultivation are:

  • Plains of North India
  • Plains of North-Eastern India
  • Coastal areas
  • Deltaic regions.

(d) Irrigated rice is produced in Punjab, Haryana, Western Uttar Pradesh and Rajasthan. Development of dense network of canals and tubewells has made it possible to grow rice in these states which receive less rainfall.

Question 20. (a) Which is the second most important cereal crop of India? Write the geographical conditions required for its growth. (b) Name two major zones of wheat growth and name six major states of wheat production. Answer: (a) Wheat is the second most important cereal crop of India. It is the main food crop. Geographical conditions:

  • Cool and moist growing season.
  • Bright sunshine at the time of ripening.
  • Rainfall: 50 to 75 cm evenly distributed over the growing season.
  • Loamy soil is best for its growth.

(b) Two important wheat growing zones in India are:

  • The Ganga-Sutlej plains in the North-West and
  • Black soil region of the Deccan.

The major wheat producing states are: Punjab, Haryana, Uttar Pradesh, Bihar, Rajasthan and Madhya Pradesh.

Question 21. Which is the third most important food crop of India? Name the cropping season in which it is grown. Name four leading states of its production. Answer: Jowar is the third most important food crop with respect to area and production. It is mainly the crop of Kharif season. It is a rainfed crop grown in moist areas. Leading states of production are: Maharashtra, Karnataka, Andhra Pradesh and Madhya Pradesh.

Question 22. What are coarse grains? Why are they important in India? Name the crops which are included in this category and name three leading states producing each of these crops. Answer: Millets are called coarse grains. They are important because they have high nutritional value and make an important part of the diet for poor people. Most important millets are as follows:

  • Ragi —Leading producer is Karnataka, followed by Tamil Nadu. Himachal Pradesh, Jharkhand, Uttaranchal, Sikkim etc are other important regions.
  • Jowar—Maharashtra is the leading producer followed by Karnataka, Andhra Pradesh, Madhya Pradesh.
  • Bajra—It grows well on sandy soils and shallow black soils. Rajasthan is the largest producer followed by Uttar Pradesh, Maharashtra, Gujarat and Haryana.

Question 23. Write two geographical conditions required for the growth of Ragi and write its nutritional value. Answer: Geographical conditions:

  • Ragi grows well in dry regions.
  • It grows well on red, black, sandy, loamy and shallow black soils.

Nutritional value: Ragi is rich in iron, calcium, other micro-nutrients and roughage.

Question 24. Mention two geographical conditions required for the growth of Maize crop in India. Describe three factors which have contributed to increase maize production. Write four major maize producing states. (2012) Answer: Geographical conditions required for the growth of maize crop in .India:

  • It is a kharif crop which requires temperature between 21° C to 27° C. It requires moderate rainfall between 50-100 cm.
  • use of modern inputs such as HYV seeds;
  • use of fertilisers; and
  • use of irrigation facilities.
  • major maize producing state: Karnataka, Uttar Pradesh, Bihar, Andhra Pradesh and Madhya Pradesh.

Question 25. (a) Name three pulses each of Rabi and Kharif season. Write their importance for human beings and for agriculture. (b) What is India’s position in the world with regard to the production of pulses? Name five leading states producing pulses. (2015) Answer: (a) Pulses of Rabi season: Tur (arhar), urad, moong. Pulses of Kharif season: Masur, peas, gram. Importance of pulses:

  • For agriculture. Being leguminous crops, they help in restoring soil fertility by utilising nitrogen from the air (nitrogen fixation). Therefore, these are mostly grown in rotation with other crops.
  • They need less moisture and survive even in dry conditions.

(b) India is the largest producer of pulses in the world. Major pulse producing states are: Madhya Pradesh, Uttar Pradesh, Rajasthan, Maharashtra and Karnataka.

Question 26. (a) What is India’s position in the world regarding sugarcane production? Write the geographical conditions required for its growth. (b) Write the major states that produce sugarcane. (c) Name four products obtained from sugarcane. Answer: (a) India is the second largest producer of sugarcane after Brazil. Geographical conditions: It is a tropical as well as subtropical crop.

  • It grows well in hot and humid climate.
  • Temperature: 21°C to 27°C.
  • Annual rainfall between 75 cm and 100 cms. Irrigation is required in the regions of low rainfall.
  • It can be grown on a variety of soils.
  • It needs manual labour from sowing to harvesting.

(b) Six major states producing sugarcane are: Uttar Pradesh, Maharashtra, Karnataka, Tamil Nadu, Andhra Pradesh, Bihar, Punjab and Haryana.

(c) Sugarcane is the main source of sugar, gur (jaggery), khandsari and molasses.

Question 27. (a) What percentage of the total cropped area of India is under oilseed production? What is India’s position in the world with regard to oilseed production? (b) Name six oilseeds produced in India. What are their main uses? (c) Which oilseed constitutes 50% of the oilseeds produced in the country? Name three major states producing this oilseed. (d) Name three oilseeds of Kharif season and three of Rabi season. Answer: (a) India is the largest producer of oilseeds in the world. 12% of the total cropped area is under oilseed production

(b) Six major oilseeds produced in India are: Groundnut, mustard, coconut, sesamum (til), soyabean, castor-seeds, linseed, sunflower and cotton-seeds. Most of these oilseeds are edible and are used as cooking mediums. Some are also used as raw material in the production of soap, cosmetics and ointments.

(c) Groundnut is a Kharif crop and constitutes 50% share in the total oilseed production. Andhra Pradesh, Tamil Nadu, Karnataka, Gujarat and Maharashtra are important producers of groundnut.

Oilseeds of Kharif season Oilseeds of Rabi season
Groundnut Mustard
Sesamum in North India Linseed
Castor-seed Sesamum in South India

Question 28. Write the geographical and labour conditions required for the growth of tea. Answer: Geographical conditions:

  • It grows well in tropical and subtropical climates.
  • It requires deep, fertile, well drained soil, rich in humus and organic matter.
  • It requires warm and moist frost free climate round the year.
  • Frequent showers evenly distributed through the year ensure continuous growth of tender leaves.
  • Tea is a labour intensive industry. It requires abundant, cheap and skilled labour.
  • It is processed within tea gardens to restore its freshness.

Question 29. What is India’s position in the world regarding tea production? Name three states each which produce tea in: (i) North-Eastern India (ii) Himalayan states/parts of the states (iii) Peninsular States. Answer: India is the leading producer and exporter of tea in the world. Three states producing tea in each of the following:

North-Eastern States Himalayan Regions/States Peninsular States
1. Assam 1. Hills of West Bengal 1. Tamil Nadu
2. Meghalaya (Darjeeling & Jalpaiguri Districts) 2. Kerala
3. Tripura 2. Himachal Pradesh 3. Andhra Pradesh
3. Uttaranchal

Question 30. Which is the most important beverage crop of Southern India and what is its percentage share in the world? Where did its cultivation initially start in India? Name three major states of its production. Answer: Coffee is the most important beverage crop of Southern India. India produces about 4% of the world’s coffee production. Three major states which produce coffee are: Karnataka, Kerala and Tamil Nadu, mainly in Nilgiri Hills. Initially its cultivation started in Baba Budan Hills.

Question 31. Name a good variety of coffee which is produced in India and name the country from where it was initially brought. Why Indian coffee has great international demand? Answer: Arabica, the good variety of coffee was originally brought from Yemen. Indian coffee is in great demand because this coffee is of a very good quality.

Question 32. What is horticulture? (2012) Answer: Cultivation of fruits, vegetables and flowers is called horticulture.

Question 33. What is India’s position in the world in the production of fruits and vegetables? Name the different Indian fruits which are in great demand world over and also name the states where each is produced. Answer: India is the largest producer of fruits and vegetables in the world. Mangoes grow in Maharashtra, Andhra Pradesh, Uttar Pradesh and West Bengal. Bananas grow well in Kerala, Mizoram, Maharashtra and Tamil Nadu. Oranges grow well in Nagpur and Cherrapunjee (Meghalaya). Lichi and Guavas grow in Uttar Pradesh and Bihar. Grapes grow well in Andhra Pradesh and Maharashtra. Pineapple grows well in Meghalaya. Apples, Pears, Apricots and Walnuts grow well in Jammu and Kashmir and Himachal Pradesh.

Question 34. Name four temperate fruits which are produced in India and name the states which grow them in abundance. Answer: Four temperate crops are: apples, pears, apricots and walnuts. Himachal Pradesh and Jammu & Kashmir are the leading producers of these temperate fruits.

Question 35. Describe India’s position in vegetable production. Answer:

  • India produces about 13% of the world’s vegetables.
  • It stands first in the production of peas and cauliflower.
  • It stands second in the production of onions, cabbage, tomatoes and brinjal and stands fourth in the production of potatoes.

Question 36. Name three non-food crops and write three major states of their production respectively. Answer: The three non-food crops are: rubber, cotton and jute. Rubber is produced in Kerala, Tamil Nadu and Karnataka, Andaman and Nicobar islands and Garo hills of Meghalaya. Cotton is mainly produced in Maharashtra, Gujarat and Madhya Pradesh. Jute is mainly grown in West Bengal, Bihar and Assam.

Question 37. Write the geographical conditions required for the growth of rubber. Name the item which consumes maximum share of rubber for its manufacture. Answer: Conditions:

  • It requires hot and humid climate.
  • Rainfall – 200 cms.
  • Temperature – above 25°C.

Maximum rubber is consumed in the manufacture of auto tyres and tubes and cycle tyres and tubes. They together consume about 57.8% of the total rubber production.

Question 38. Name four major fibre crops grown in India. What is ‘sericulture’? Answer: Cotton, jute, hemp and natural silk are the four major fibre crops grown in India. The first three are derived directly from the crops grown in the soil, but silk is obtained from the cocoons of silk worms fed on mulberry leaves. Sericulture: Rearing of silk worms for the production of silk fibre is known as sericulture.

Question 39. Give an account of fibre crop which is mainly grown in Deccan Plateau region under the following heads: (i) Its position in the world production (ii) geographical conditions and (iii) major states of production. Answer: Cotton is the fibre crop which is mainly grown in the black soil of the Deccan Plateau region. (i) Position. India is the 3rd largest producer of cotton in the world. (ii) Geographical conditions. Cotton requires:

  • high temperature.
  • light rainfall or irrigation.
  • 210 frost-free days.
  • bright sunshine for its growth.
  • black cotton soil which is very good for its growth.
  • It is a Kharif crop and requires 6-8 months to mature.

(iii) Major cotton producing states are. Maharashtra, Gujarat, Madhya Pradesh, Karnataka, Andhra Pradesh, Tamil Nadu, Punjab, Haryana and Uttar Pradesh.

Question 40. Which fibre crop is called as the ‘Golden fibre’? Which is the most important region of its growth and why? Give one major reason why it is losing the market now. Answer: Jute is known as the golden fibre. Jute is mainly grown in West Bengal, especially in the Hooghly Basin because there the geographical conditions favour its growth. These conditions are:

  • High temperature required during the time of growth.
  • Jute grows well on well-drained fertile soils in the flood plains where soils are renewed every year. Due to its high cost, it is losing market to synthetic fibres and other packing materials particularly to nylon.

Question 41. Name the major jute producing states and list some items or products made out of jute. Answer: Jute producing states are: West Bengal, Bihar, Assam, Orissa and Meghalaya. Jute products are: gunny bags, ropes, mats, carpets, yarns and other ornamental artefacts.

Technological And Institutional Reforms

Question 42. Give the main reasons which have necessitated agricultural reforms. Answer: Reforms in agriculture are necessary because of the following reasons:

  • Sustained uses of land without compatible techno-institutional changes have hindered the pace of agricultural development.
  • Most of the farmers still depend on monsoons because large parts of the country still do not have irrigation facilities.
  • Farmers still depend on natural fertility in order to carry on their agriculture, i.e., they lack material resources, e.g., fertilizers, etc.
  • Ours is an agricultural economy and about 63% of people depend on agriculture for employment and livelihood, therefore reforms have to be implemented.
  • For raising the agricultural production and productivity levels to produce sufficient food for the growing population.
  • To overcome environmental, economic and social constraints, agricultural reforms have to be seriously implemented.
  • The declining share in GDP is a matter of serious concern because decline and stagnation in agriculture will lead to decline in other spheres of economy.

Question 43. What are ‘Institutional Reforms’? Enlist various institutional reforms taken by the Indian Government to bring about improvements in agriculture. (2015) Answer: Institutional reforms: Steps taken by the government to bring about improvements in agriculture are termed as ‘Institutional Reforms’. Some steps are:

  • Collectivisation and consolidation of land holdings to make them economically viable.
  • The green revolution based on the use of package technology and the White Revolution to increase milk production are important strategies which were initiated to improve agriculture.
  • Cooperation with farmers and Abolition of Zamindari system.
  • Provision of crop insurance to protect the farmers against losses caused by natural calamities, i.e. drought, flood, cyclone, fire and disease.
  • Establishment of ‘Grameen Banks’, Cooperative Societies and Banks for providing loan facilities to the farmers at lower rates of interest.
  • Kissan Credit Card (KCC), Personal Accident Insurance Scheme (PAIS) are some other schemes introduced by the government for the benefit of farmers.
  • Special weather bulletins and agricultural programmes for farmers were introduced on the Radio and TV.
  • Announcement of minimum support price, remunerative and procurement prices for crops to check the exploitation of farmers by speculators and middlemen and removing the elements of uncertainty.

Question 44. Describe some concerted efforts made by the government to modernise Indian agriculture and improve its share in the GDP. Answer: The following measures have been taken by the government to modernise agriculture and improve its share in the GDP:

  • Establishment of Indian Council of Agricultural Research (ICAR).
  • Setting up of Agricultural Universities.
  • Development of advanced Veterinary Services and Animal Breeding Centres.
  • Development of horticulture.
  • Research and Development in the field of meteorology and weather forecasts, etc. Improving the rural infrastructure is essential for the same.

Question 45. “The contribution of agriculture to national economy is on the decline.” Write five facts to support this statement. Or, What is the contribution of agriculture to national economy, employment and output? Is it declining or encouraging? Answer: Contribution of agriculture is showing a declining trend:

  • The share of agriculture in GDP has shown a declining trend since 1951.
  • Yet its share in providing employment and livelihood to the population continued to be as high as 63% in 2001.
  • Although GDP growth rate of the country is increasing, it is not generating sufficient employment opportunities in the country, especially in the field of agriculture.
  • Farmers are dragging away their investment from agriculture because they have to face big challenges from international competition. This has resulted in further downfall in employment in agriculture.
  • The growth rate in agriculture is decelerating which is an alarming situation.

Question 46. What are the challenges being faced by Indian farmers? What has this resulted in? Answer: Challenges faced by Indian farmers:

  • Reduction in public investment by government in the agricultural sector particularly in areas of irrigation, power, rural roads, market and mechanisation.
  • Subsidy on fertilizers has decreased leading to increase in the cost of production.
  • Reduction in import duties on agricultural products has proved detrimental to agriculture in the country. All these factors have led to stiff international competition. Farmers are thus withdrawing their investment from agriculture causing a downfall in agricultural employment.

Question 47. Define each of the following: (a) Green Revolution (b) White Revolution (c) Yellow Revolution (d) Blue Revolution (e) Gene Revolution. Answer: (a) Green Revolution. Agricultural Revolution which resulted in increased production of foodgrains because of the use of HYV (High Yielding Varieties) seeds, fertilizers, proper irrigation and other modem inputs. (b) White Revolution. Revolutionary increase in the production of milk which was spearheaded by Operation Flood. (c) Yellow Revolution. Increase in the output of oilseeds is termed as Yellow Revolution. (d) Blue Revolution refers to increased output of fish and fish products. (e) Gene Revolution refers to the development of genetically modified seeds for increasing the yield per hectare. These seeds are environmentally sustainable.

Question 48. Write four drawbacks of green revolution. Answer:

  • Due to overuse of chemicals land degradation has taken place.
  • Excessive irrigation caused drying of acquifers.
  • It became a cause for vanishing biodiversity.
  • It has widened the gap between poor and rich farmers because only agriculturally rich areas have benefitted from it and not the small farmers.

Question 49. (a) Give four reasons why Indian farmers should switch over from cereals to high value crops’ cultivation. (b) What will be the implication of this change? Name three countries which have successfully done this. Answer: (a) Indian farmers should switch from cereals to high value crops because of the following reasons:

  • It will improve their income.
  • It will reduce environmental degradation at the same time.
  • Fruits, medicinal herbs, bio-diesel crops, flowers and vegetables need much less irrigation than rice or sugarcane.
  • India’s wide variety of climates can be harnessed to grow a wide range of high value crops.

(b) Its implications are:

  • India has to import food.
  • If we import food while exporting high value crops, our economy will grow.
  • Three countries which have such successful economies are: Chile, Israel and Italy, which export farm products (fruits, wine, olives, speciality seeds) and import food.

Social Science Class 10 Important Questions

Free resources.

NCERT Solutions

Quick Resources

Case Based Questions Test: Agriculture - Class 10 MCQ

15 questions mcq test - case based questions test: agriculture, read the sources given below and answer the questions that follows: source a : primitive subsistence farming this type of farming is still practised in few pockets of india. primitive subsistence agriculture is practised on small patches of land with the help of primitive tools like hoe, dao and digging sticks, and family/community labour. this type of farming depends upon monsoon, natural fertility of the soil and suitability of other environmental conditions to the crops grown. it is a 'slash and burn' agriculture. farmers clear a patch of land and produce cereals and other food crops to sustain their family. when the soil fertility decreases, the farmers shift and clear a fresh patch of land for cultivation. this type of shifting allows nature to replenish the fertility of the soil through natural processes; land productivity in this type of agriculture is low as the farmer does not use fertilisers or other modern inputs. it is known by different names in different parts of the country. source b : commercial farming plantation is also a type of commercial farming. in this type of farming, a single crop is grown on a large area. the plantation has an interface of agriculture and industry. plantations cover large tracts of land, using capital intensive inputs, with the help of migrant labourers. all the produce is used as raw material in respective industries. in india, tea, coffee, rubber, sugarcane, banana, etc., are important plantation crops. tea in assam and north bengal coffee in karnataka are some of the important plantation crops grown in these states. since the production is mainly for market, a well-developed network of transport and communication connecting the plantation areas, processing industries and markets plays an important role in the development of plantations. answer the following mcqs by choosing the most appropriate option. primitive subsistence farming depends on:.

  • B. Fertility of Soil
  • C. Environmental Conditions
  • D. All of the above

case study of agriculture class 10

Read the sources given below and answer the questions that follows: SOURCE A : Primitive Subsistence Farming This type of farming is still practised in few pockets of India. Primitive subsistence agriculture is practised on small patches of land with the help of primitive tools like hoe, dao and digging sticks, and family/community labour. This type of farming depends upon monsoon, natural fertility of the soil and suitability of other environmental conditions to the crops grown. It is a 'slash and burn' agriculture. Farmers clear a patch of land and produce cereals and other food crops to sustain their family. When the soil fertility decreases, the farmers shift and clear a fresh patch of land for cultivation. This type of shifting allows Nature to replenish the fertility of the soil through natural processes; land productivity in this type of agriculture is low as the farmer does not use fertilisers or other modern inputs. It is known by different names in different parts of the country. SOURCE B : Commercial Farming Plantation is also a type of commercial farming. In this type of farming, a single crop is grown on a large area. The plantation has an interface of agriculture and industry. Plantations cover large tracts of land, using capital intensive inputs, with the help of migrant labourers. All the produce is used as raw material in respective industries. In India, tea, coffee, rubber, sugarcane, banana, etc., are important plantation crops. Tea in Assam and North Bengal coffee in Karnataka are some of the important plantation crops grown in these states. Since the production is mainly for market, a well-developed network of transport and communication connecting the plantation areas, processing industries and markets plays an important role in the development of plantations. Answer the following MCQs by choosing the most appropriate option. When the ____________ decreases, the farmers clear a fresh patch of land for cultivation:

  • A. Production
  • B. Soil fertility
  • C. Natural Calamities
  • D. Industries
1 Crore+ students have signed up on EduRev. Have you?

Read the sources given below and answer the questions that follows: SOURCE A : Primitive Subsistence Farming This type of farming is still practised in few pockets of India. Primitive subsistence agriculture is practised on small patches of land with the help of primitive tools like hoe, dao and digging sticks, and family/community labour. This type of farming depends upon monsoon, natural fertility of the soil and suitability of other environmental conditions to the crops grown. It is a 'slash and burn' agriculture. Farmers clear a patch of land and produce cereals and other food crops to sustain their family. When the soil fertility decreases, the farmers shift and clear a fresh patch of land for cultivation. This type of shifting allows Nature to replenish the fertility of the soil through natural processes; land productivity in this type of agriculture is low as the farmer does not use fertilisers or other modern inputs. It is known by different names in different parts of the country. SOURCE B : Commercial Farming Plantation is also a type of commercial farming. In this type of farming, a single crop is grown on a large area. The plantation has an interface of agriculture and industry. Plantations cover large tracts of land, using capital intensive inputs, with the help of migrant labourers. All the produce is used as raw material in respective industries. In India, tea, coffee, rubber, sugarcane, banana, etc., are important plantation crops. Tea in Assam and North Bengal coffee in Karnataka are some of the important plantation crops grown in these states. Since the production is mainly for market, a well-developed network of transport and communication connecting the plantation areas, processing industries and markets plays an important role in the development of plantations. Answer the following MCQs by choosing the most appropriate option. ______ crop is grown in Commercial farming:

Read the sources given below and answer the questions that follows:

SOURCE A : Primitive Subsistence Farming

This type of farming is still practised in few pockets of India. Primitive subsistence agriculture is practised on small patches of land with the help of primitive tools like hoe, dao and digging sticks, and family/community labour. This type of farming depends upon monsoon, natural fertility of the soil and suitability of other environmental conditions to the crops grown. It is a 'slash and burn' agriculture. Farmers clear a patch of land and produce cereals and other food crops to sustain their family. When the soil fertility decreases, the farmers shift and clear a fresh patch of land for cultivation. This type of shifting allows Nature to replenish the fertility of the soil through natural processes; land productivity in this type of agriculture is low as the farmer does not use fertilisers or other modern inputs. It is known by different names in different parts of the country.

SOURCE B : Commercial Farming

Plantation is also a type of commercial farming. In this type of farming, a single crop is grown on a large area. The plantation has an interface of agriculture and industry. Plantations cover large tracts of land, using capital intensive inputs, with the help of migrant labourers. All the produce is used as raw material in respective industries. In India, tea, coffee, rubber, sugarcane, banana, etc., are important plantation crops. Tea in Assam and North Bengal coffee in Karnataka are some of the important plantation crops grown in these states. Since the production is mainly for market, a well-developed network of transport and communication connecting the plantation areas, processing industries and markets plays an important role in the development of plantations.

Answer the following MCQs by choosing the most appropriate option.

In commercial farming, we used:

  • A. HYV seeds
  • B. Old methods of inputs
  • C. Small patches of land
  • D. Skilled labour

U SOURCE A : Wheat

This is the second most important cereal crop. It is the main food crop, in north and north-western part of the country. This rabi crop requires a cool growing season and a bright sunshine at the time of ripening. It requires 50 to 75 cm of annual rainfall evenly-distributed over the growing season. There are two important wheat-growing zones in the country – the Ganga-Satluj plains in the northwest and black soil region of the Deccan. The major wheat-producing states are Punjab, Haryana, Uttar Pradesh, Bihar, Rajasthan and parts of Madhya Pradesh.

Rice It is the staple food crop of a majority of the people in India. Our country is the second largest producer of rice in the world after China. It is a kharif crop which requires high temperature, (above 25°C) and high humidity with annual rainfall above 100 cm. In the areas of less rainfall, it grows with the help of irrigation.

How much annual rainfall is required for Wheat?

  • A. 50 to 75 cm
  • B. below 50 cm
  • C. Above 100 cm
  • D. 75 to 150 cm

Wheat is also grown in areas having lesser amount of rainfall, i.e., 25 cm. This has been done by adopting dry farming method. Also where irrigation facil­ities are available, wheat is cultivated in dry lands also.

SOURCE B : Rice

It is the staple food crop of a majority of the people in India. Our country is the second largest producer of rice in the world after China. It is a kharif crop which requires high temperature (above 25°C) and high humidity with annual rainfall above 100 cm. In the areas of less rainfall, it grows with the help of irrigation.

What is climatic conditions required for Rice?

  • A. High rainfall
  • B. High temperature
  • C. High humidity

It is the staple food crop of a majority of the people in India. Our country is the second largest producer of rice in the world after China. It is a kharif crop which requires high temperature, (above 25°C) and high humidity with annual rainfall above 100 cm. In the areas of less rainfall, it grows with the help of irrigation. Answer the following MCQs by choosing the most appropriate option.

Wheat growing zones in India :

  • A. The Ganga-Satluj plains in the north-west
  • B. Black soil region of the Deccan plateau
  • C. Both 'a' and 'b'
  • D. None of the above

_________ is the largest producer of Rice.

  • D. Australia

Read the extract given below and answer the questions that follows:

Mahatma Gandhi declared Vinoba Bhave as his spiritual heir. He also participated in Satyagraha as one of the foremost satyagrahis. He was one of the votaries of Gandhi’s concept of gram swarajya. After Gandhi's martyrdom, Vinoba Bhave undertook padyatra to spread Gandhiji’s message that covered almost the entire country. Once, when he was delivering a lecture at Pochampally in Andhra Pradesh, some poor landless villagers demanded some land for their economic well-being. Vinoba Bhave could not promise it to them immediately but assured them to talk to the Government of India regarding provision of land for them if they undertook cooperative farming. Suddenly, Shri Ram Chandra Reddy stood up and offered 80 acres of land to be distributed among 80 landless villagers. This act was known as ‘Bhoodan’. Later he travelled and introduced his ideas widely all over India. Some zamindars, owners of many villages, offered to distribute some villages among the landless. It was known as Gramdan. However, many land-owners chose to provide some part of their land to the poor farmers due to the fear of land ceiling act. This Bhoodan-Gramdan movement initiated by Vinoba Bhave is also known as the Bloodless Revolution.

Who declared Vinoba Bhave as his spiritual heir?

  • B. Rabindra Nath Tagore
  • C. Mahatma Gandhi
  • D. All of the Above

The act done by Shri Ram Chandra Reddy was called _________ .

  • C. Annaprasana
  • D. Atmasamman

Some zamindars, owners of many villages, offered to distribute some villages among the landless. What was it known as?

  • D. None of these

Mahatma Gandhi declared Vinoba Bhave as his spiritual heir. He also participated in Satyagraha as one of the foremost satyagrahis. He was one of the votaries of Gandhi’s concept of gram swarajya. After Gandhi's martyrdom, Vinoba Bhave undertook padyatra to spread Gandhiji’s message that covered almost the entire country. Once, when he was delivering a lecture at Pochampally in Andhra Pradesh, some poor landless villagers demanded some land for their economic well-being. Vinoba Bhave could not promise it to them immediately but assured them to talk to the Government of India regarding provision of land for them if they undertook cooperative farming. Suddenly, Shri Ram Chandra Reddy stood up and offered 80 acres of land to be distributed among 80 landless villagers. This act was known as ‘Bhoodan’. Later he travelled and introduced his ideas widely all over India. Some zamindars, owners of many villages, offered to distribute some villages among the landless. It was known as Gramdan. However, many land-owners chose to provide some part of their land to the poor farmers due to the fear of land ceiling act. This Bhoodan-Gramdan movement initiated by Vinoba Bhave is also known as the Bloodless Revolution.

Where did Vinoba Bhave delivered his lecture for villagers well- being?

Pochampalli in Andhra Pradesh

Warangal in Andhra Pradesh

Panchvati in Himachal Pradesh

Sirmaur in Himachal Pradesh

After Gandhiji's martyrdom, Vinoba Bhave undertook padyatra to spread Gandhiji's message that covered almost the entire country. Once, when he was delivering a lecture at Pochampalli in Andhra Pradesh, some poor landless villagers demanded some land for their economic well-being.

Tea cultivation is an example of plantation agriculture. It is also an important beverage crop introduced in India initially by the British. Today, most of the tea plantations are owned by Indians. The tea plant grows well in tropical and sub tropical climates endowed with deep and fertile well-drained soil, rich in humus and organic matter. Tea bushes require warm and moist frost free climate all through the year. Frequent showers evenly distributed over the year ensure continuous growth of tender leaves. Tea is a labour-intensive industry. It requires abundant, cheap and skilled labour. Tea is processed within the tea garden to restore its freshness. Major tea producing states are Assam, hills of Darjeeling and Jalpaiguri districts, West Bengal, Tamil Nadu and Kerala. Apart from these, Himachal Pradesh, Uttarakhand, Meghalaya, Andhra Pradesh and Tripura are also tea-producing states in the country. In 2015 India was the second largest producer of tea after China.

Who introduced tea cultivation in India?

Which of the following states is the largest producer of tea?

  • B. West Bengal
  • C. Uttarakhand

Tea cultivation is an example of plantation agriculture. It is also an important beverage crop introduced in India initially by the British. Today, most of the tea plantations are owned by Indians. The tea plant grows well in tropical and subtropical climates endowed with deep and fertile well-drained soil, rich in humus and organic matter. Tea bushes require a warm and moist frost free climate all through the year. Frequent showers evenly distributed over the year ensure continuous growth of tender leaves. Tea is a labour-intensive industry. It requires abundant, cheap and skilled labour. Tea is processed within the tea garden to restore its freshness. Major tea producing states are Assam, hills of Darjeeling and Jalpaiguri districts, West Bengal, Tamil Nadu and Kerala. Apart from these, Himachal Pradesh, Uttarakhand, Meghalaya, Andhra Pradesh and Tripura are also tea-producing states in the country. In 2015 India was the second largest producer of tea after China.

Which of the following countries was the largest production of tea in 2015?

India 

China 

--> and get INR 200 additional OFF

Top Courses for Class 10

case study of agriculture class 10

Important Questions for Case Based Questions Test: Agriculture

Case based questions test: agriculture mcqs with answers, online tests for case based questions test: agriculture.

cation olution
Join the 10M+ students on EduRev

Welcome Back

Create your account for free.

case study of agriculture class 10

Forgot Password

Agriculture

Class 10 - ncert geography contemporary india 2 solutions, multiple choice questions.

Which one of the following describes a system of agriculture where a single crop is grown on a large area?

  • Shifting Agriculture

Plantation Agriculture

  • Horticulture
  • Intensive Agriculture

Which one of the following is a rabi crop?

Which one of the following is a leguminous crop?

Answer in about 30 words

Name one important beverage crop and specify the geographical conditions required for its growth.

Geographical conditions — It requires tropical and sub-tropical climate endowed with deep and fertile well-drained soil, rich in humus and organic matter. It grows well in warm, moist and frost-free climate all through the year.

Name one staple crop of India and the regions where it is produced.

Rice is one of the staple crop of India. It is grown in plains of north and north-eastern India, coastal areas and the deltaic regions. It is also grown in Punjab, Haryana, western Uttar Pradesh and parts of Rajasthan with the help of irrigation.

Enlist the various institutional reform programmes introduced by the government in the interest of farmers.

  • Land reforms and land development programme.
  • Provision for crop insurance.
  • Establishment of Grameen banks, cooperative societies.
  • Kissan Credit Card (KCC) and Personal Accident Insurance Scheme (PAIS).
  • Special Weather bulletins and agricultural programmes on radio and television.
  • Minimum support price (MSP).

Answer in about 120 words

Suggest the initiative taken by the government to ensure the increase in agricultural production.

The the initiatives taken by the government to ensure the increase in agricultural production are:

  • Collectivisation, consolidation of holdings, cooperation and abolition of zamindari.
  • Land Reform was the main focus of the 'First Five Year Plan'.
  • The Green Revolution and the White Revolution were initiated to improve the Indian agriculture.
  • Provision for crop insurance and establishment of Grameen banks, cooperative societies and banks for providing loan facilities to the farmers at lower rates of interest.
  • Minimum support price (MSP) and subsidy on agricultural inputs and resources such as power and fertilizers.

Describe the geographical conditions required for the growth of rice.

Geographical conditions required for the growth of rice are:

  • Rice is a kharif crop which requires hot and humid climate for cultivation.
  • It requires high temperature above 25°C.
  • High humidity with annual rainfall above 100 cm are favourable for the growth of rice.
  • It is grown in the plains of north and north-eastern India, coastal areas and the deltaic regions.
  • In the areas of less rainfall like Punjab, Haryana and western Uttar Pradesh and parts of Rajasthan, it grows with the help of irrigation.
  • Rice cultivation requires flat terrain because it needs standing water through out most of its life cycle.

Talk to our experts

1800-120-456-456

Agriculture Class 10 Notes CBSE Geography Chapter 4 (Free PDF Download)

  • Revision Notes
  • Social Science: Geography
  • Chapter 4 Agriculture

ffImage

Agriculture Class 10 Notes Geography Chapter 4 - PDF Download

Class 10 Geography Chapter 4 is Agriculture. Agriculture engages two-third of India’s population and is the primary activity which produces raw material for a wide range of industries. In the Agriculture notes, you will study the different types of farming methods, major crops grown in India, different cropping patterns and contribution of agriculture to the nation’s economy, employment and output. Chapter 4 Geography class 10 notes will give an insight into the agricultural sector of the country. Class 10 agriculture is a very important chapter as it gives you an idea about this important economic activity. Notes of Agriculture class 10 will also help you prepare for your board examinations.

Topics Covered in the Chapter 4 Agriculture of Class 10

Below given is the list of various topics discussed in the chapter ‘Agriculture’. 

Types of Farming

Primitive Subsistence Farming

Intensive Subsistence Farming 

Commercial Farming 

Cropping Pattern

Major Crops (Wheat, Millets, Maize, Pulses)

Food Crops other than Grains (Sugarcane, Oil Seeds, Tea, Coffee, Horticulture Crops)

Non-Food Crops (Rubber, Fibre Crops, Cotton, Jute)

Technological and Institutional Reforms

Contribution of agriculture to the national economy, employment and output.

Impact of Globalisation on Agriculture  

Highlights of the Chapter: Agriculture

Board

Cental Board of Secondary Education (CBSE)

Textbook

Contemporary India-2

Class

10

Subject

Social Science (Geography)

Chapter

4

Chapter Name

Agriculture

Category

Revision Notes

icon

Related Chapters

Arrow-right

Access Class 10 Social Science Chapter 4: Agriculture Notes

Agriculture, an age-old economic activity. About two-thirds of our population is engaged in agriculture. Agriculture not only produces grains but also raw materials for many industries.

Farming varies from subsistence to commercial type.

At present the following farming systems are practiced:

1. Primitive Subsistence Farming

It is also called slash and burn agriculture.

This type of farming is majorly practiced on small patches, using primitive tools like hoe, dao and digging sticks. Family and community labor are involved in this type of farming. 

Nature plays a significant role in this type of farming. The production depends upon fertility, monsoon and suitability of other environmental conditions.

First, a patch of land is cleared and then food crops are grown on it. After the soil fertility decreases, they shift to another patch which allows the previous patch to replenish the nutrients and new fertile land for plantation.

No modern equipment or advanced farming techniques are used in this type of farming.

2. Intensive Subsistence Farming

Usually practiced in areas where there is high pressure of production on land. Various biochemical inputs and modern irrigation techniques are used to aid production.

3. Commercial Farming

High doses of high yielding variety (HYV) seeds, fertilizers, insecticides and 

pesticides are used.

Rice is a commercial crop in Haryana.

Another type of commercial farming is plantation in which a single crop is grown over a large area.

In India, tea, coffee, rubber, sugarcane, banana, etc.. are important plantation crops. Tea in Assam and North Bengal coffee in Karnataka are some of the important plantation crops grown in these states.

Cropping Patterns

Three cropping seasons:

Rabi crops are sown in winter from October to December and harvested in summer from April to June.

Wheat, barley, peas, gram, and mustard are major rabi crops.

states from the north and northwestern parts such as Punjab, Haryana, Himachal Pradesh, Jammu and Kashmir, Uttarakhand and Uttar Pradesh grow most quantities of wheat and other rabi crops.

Kharif crops are grown with the onset of monsoon and are harvested in September-October.

Important crops grown during this season are paddy, maize, jowar, bajra, tur (arhar), moong, urad, cotton, jute, groundnut and soyabean. 

Some of the most important rice-growing regions are Assam, West Bengal, coastal regions of Odisha, Andhra Pradesh, Telangana, Tamil Nadu, Kerala and Maharashtra, particularly the (Konkan coast) along with Uttar Pradesh and Bihar.

A short season during the summer months, in between the rabi and the Kharif seasons is known as the Zaid season.

Watermelon, muskmelon, cucumber, vegetables and fodder crops are some of the major zaid crops.

Major Crops

India is the second-largest producer of rice in the world after China.

It is a Kharif crop and requires elevated temperature, (above 25°C) and high humidity with annual rainfall above 100 cm.

Major rice production is found in the plains of north and north-eastern India, coastal areas, and the deltaic regions.

It requires 50 to 75 cm of annual rainfall.

There are two important wheat-growing zones in the country – the Ganga-Satluj plains in the northwest and the black soil region of the Deccan.

The major wheat-producing states are Punjab, Haryana, Uttar Pradesh, Bihar, Rajasthan, and parts of Madhya Pradesh

Jowar, bajra and ragi are the important millets grown in India.

Jowar is a rain-fed crop cultivated in Maharashtra, Karnataka, Andhra Pradesh and Madhya Pradesh

Bajra grows well on sandy soils and shallow black soil. 

Major Bajra producing States were: Rajasthan, Uttar Pradesh, Maharashtra, Gujarat and Haryana.

Ragi grows well on dry red, black, sandy, loamy and shallow black soils. 

Major ragi producing states are: Karnataka, Tamil Nadu, Himachal Pradesh, Uttarakhand, Sikkim, Jharkhand and Arunachal Pradesh.

It is a Kharif crop and requires a temperature between 21°C to 27°C.

Grows well in old alluvial soil. 

Major maize-producing states are Karnataka, Uttar Pradesh, Bihar, Andhra Pradesh, Telangana and Madhya Pradesh.

Major pulses cultivated are tur (arhar), urad, moong, masur, and peas.

They help in restoring soil fertility as there are leguminous crops and nitrogen fixation. These crops are grown in rotation with cereal crops.

Major pulse producing states in India are Madhya Pradesh, Uttar Pradesh, Rajasthan, Maharashtra and Karnataka.

Food Crops other than Grains

1. sugarcane.

It is a tropical as well as a subtropical crop. 

It grows well in a hot and humid climate and 21°C to 27°C as temperature

Rainfall between 75cm and 100cm is required for its proper cultivation.

India is the second-largest producer of sugarcane, after Brazil.

Uttar Pradesh, Maharashtra, Karnataka, Tamil Nadu, Andhra Pradesh, Telangana, Bihar, Punjab and Haryana are major producers of sugarcane.

2. Oil Seeds

In 2008 India was the second largest producer of groundnut in the world after China. 

12 percent of the total cropped area of the country is used for the cultivation of various oil crops.

Main oil-seeds produced in India are groundnut, mustard, coconut, sesamum (til), soyabean, castor seeds, cotton seeds, linseed, and sunflower.

Groundnut is a kharif crop.

Gujarat was the largest producer of groundnut followed by Andhra Pradesh and Tamil Nadu. 

Linseed and mustard are rabi crops. 

Sesamum is a Kharif crop in the north and a rabi crop in south India. 

Castor seed is grown both as rabi and kharif crops.

Tea is a labour-intensive industry.

Assam, hills of Darjeeling and Jalpaiguri districts, West Bengal, Tamil Nadu and Kerala are major tea producing states in India. Apart from these, Himachal Pradesh, Uttarakhand, Meghalaya, Andhra Pradesh and Tripura are also tea-producing states in the country. 

India was the third-largest producer of tea after China and Turkey in 2008.

In 2008 India produced 3.2 per cent of the world coffee production.

It is cultivated in the Nilgiris in Karnataka, Kerala, and Tamil Nadu.

5. Horticulture Crops

In 2008 India was the second largest producer of fruits and vegetables in the world after China.

Mangoes of Maharashtra, Andhra Pradesh, Telangana, Uttar Pradesh and West Bengal, oranges of Nagpur and Cherrapunjee (Meghalaya), bananas of Kerala, Mizoram, Maharashtra and Tamil Nadu, lichi and guava of Uttar Pradesh and Bihar, pineapples of Meghalaya, grapes of Andhra Pradesh, Telangana and Maharashtra, apples, pears, apricots and walnuts of Jammu and Kashmir and Himachal Pradesh are in great demand the world over.

India produces 13 per cent of the world’s vegetables. 

Non-food Crops

It grows in a moist and humid climate with rainfall of more than 200 cm and temperature above 25°C.

It is grown in Kerala, Tamil Nadu, Karnataka and Andaman and Nicobar Islands and Garo hills of Meghalaya

India ranked fourth among the world’s natural rubber producers.

2. Fiber Crop

It is obtained from cocoons of the silkworms that feeds on mulberry leaves. 

The rearing of silkworms to produce silk fibre is known as sericulture.

India was the second-largest producer of cotton after China in 2008

It grows well in drier parts of the black cotton soil of the Deccan plateau.

It requires elevated temperature, light rainfall, and irrigation, 210 frost-free days and bright sunshine.

It is a Kharif crop and requires 6 to 8 months to mature. 

Major states are– Maharashtra, Gujarat, Madhya Pradesh, Karnataka, Andhra Pradesh, Telangana, Tamil Nadu, Punjab, Haryana, and Uttar Pradesh.

It is known as golden fibre.

Elevated temperature is required for its growth.

West Bengal, Bihar, Assam, Odisha and Meghalaya are the major jute producing states. 

The main focus of the First Five Year Plan by the government was ‘land reform’. 

The Government of India introduced agricultural reforms in the 1960s and 1970s to improve agriculture.

The Green Revolution and the White Revolution (Operation Flood) were some of the strategies adopted to improve Indian agriculture.

In the 1980s and 1990s, a comprehensive land development programme was initiated, which included both institutional and technical reforms. 

Important steps like provision for crop insurance against drought, flood, cyclone, fire and disease, the establishment of Grameen banks, cooperative societies and banks for providing loan facilities to the farmers at lower rates of interest.

Kissan Credit Card (KCC), Personal Accident Insurance Scheme (PAIS) are some other schemes introduced by the Government of India for the benefit of the farmers

The government also announces minimum support price, remunerative and procurement prices for important crops which helps to increase farmer’s profit and promotes them to grow more crops.

In 2010-11 about 52 per cent of the total workforce in India was employed under the farming sector. 

More than half of the Indian population is dependent on agriculture for their sustenance

The establishment of the Indian Council of Agricultural Research (ICAR), agricultural universities, veterinary services and animal breeding centres, horticulture development, research, and development in the field of meteorology and weather forecast, etc. were given priority for improving Indian agriculture. 

Food Security

If any segment of our population does not have this access, that segment suffers from lack of food security.

The remote areas of the country are more prone to natural disasters and uncertain food supply.

The government has resorted to two components (a) buffer stock and (b) public distribution system (PDS), to ensure food availability to all.

PDS provides food grains and other necessities at subsidized prices to the poor. 

Food Corporation of India (FCI) is responsible for procuring and stocking food grains, while distribution is ensured by public distribution system (PDS).

The FCI procures food grains from the farmers at the government announced minimum support price (MSP) and then is given to the poor at subsidized prices.

Impact of Globalization on Agriculture

After 1990, under globalization, the farmers in India have faced new challenges in the international market. 

Despite being an important producer of rice, cotton, rubber, tea, coffee, jute and spices Indian agricultural products are not able to compete with the developed countries because of the highly subsidized agriculture in those countries.

Genetic engineering is recognized as a powerful supplement in inventing new hybrid varieties of seeds that can increase production and make farming more profitable.

In fact, organic farming is much in vogue today because it is practiced without factory-made chemicals such as fertilizers and pesticides and promotes organic farming.

Important Questions and Answers

1. What are the various types of primitive subsistence farming?

Ans: It is also called slash and burn agriculture.

It is practiced in the states of Assam, Meghalaya, Nagaland and Mizoram.

‘Khil’ is primitive agriculture practiced in the Himalayan region.

‘Kumari’ is practiced in the Western Ghats.

‘Bewar’ is a type of slash and burn agriculture practiced in Madhya Pradesh.

2. What are the various cropping patterns in India? Write briefly about them.

Ans: There are three major cropping patterns in India. They are mentioned below:

1.  Rabi

  2. Kharif

3. What are the requirements to grow sugarcane? What are the regions where it is grown?

Ans: It is a tropical as well as a subtropical crop. Its various requirements are: 

India is the second-largest producer of sugarcane, after Brazil. Uttar Pradesh, Maharashtra, Karnataka, Tamil Nadu, Andhra Pradesh, Telangana, Bihar, Punjab, and Haryana are the major producers of sugarcane.

4. Write a note on rice cultivation in the country.

Ans: Rice is a staple food in India, especially Southern India.

Major rice production is found in the plains of north and north-eastern India, coastal areas, and the deltaic regions since it requires a lot of water for irrigation.

There are three paddy crops in a year namely: Aman, Aus and Boro.

Major rice producing states are Uttar Pradesh, Punjab, Haryana, and Rajasthan.

5. Write a short note on oilseeds.

Ans: In 2008 India was the second-largest producer of groundnut in the world after China. 12 percent of the total cropped area of the country is used for the cultivation of various oil crops. The main oil-seeds produced in India are groundnut, mustard, coconut, sesamum (til), soyabean, castor seeds, cotton seeds, linseed, and sunflower. Groundnut is a kharif crop. Gujarat was the largest producer of groundnut followed by Andhra Pradesh and Tamil Nadu. Linseed and mustard are rabi crops. Sesamum is a Kharif crop in the north and a rabi crop in south India. Castor seed is grown both as rabi and Kharif crops.

6. Write about the various horticulture crops and their region of growth.

Ans: In 2008 India was the second-largest producer of fruits and vegetables in the world after China. Mangoes of Maharashtra, Andhra Pradesh, Telangana, Uttar Pradesh and West Bengal, oranges of Nagpur and Cherrapunjee (Meghalaya), bananas of Kerala, Mizoram, Maharashtra, and Tamil Nadu, litchi and guava of Uttar Pradesh and Bihar, pineapples of Meghalaya, grapes of Andhra Pradesh, Telangana and Maharashtra, apples, pears, apricots and walnuts of Jammu and Kashmir, and Himachal Pradesh are in great demand the world over. India produces 13 per cent of the world’s vegetables.

7. What is Bhoodan and Gramdan?

Ans: Vinoba Bhave, a former Satyagrahi, initiated the movement. He tried to help the landless farmers in Pochampalli, Andhra Pradesh by asking for aid from the government but ended up receiving unexpected help from Shri Ram Charan Reddy who offered around 80 acres of land to the landless farmers. This act was popularly known as ‘Bhoodan.’ This step was appreciated and various zamindars also distributed some land amongst the poor and was called ‘Gramdan’.

8. What is the contribution of agriculture to the national economy, employment and national output?

Ans: In 2010-11 about 52 per cent of the total workforce in India was employed under the farming sector. More than half of the Indian population is dependent on agriculture for their sustenance. A decline in the number of people involved in agriculture can be disastrous for the Indian economy. The establishment of the Indian Council of Agricultural Research (ICAR), agricultural universities, veterinary services and animal breeding centers, horticulture development, research, and development in the field of meteorology and weather forecast, etc. were given priority for improving Indian agriculture.

9. Write a note on the Food Security System.

Ans: If any segment of our population does not have this access, that segment suffers from a lack of food security. The remote areas of the country are more prone to natural disasters and uncertain food supply. The government has resorted to two components (a) buffer stock and (b) public distribution system (PDS), to ensure food availability to all. PDS provides food grains and other necessities at subsidized prices to the poor.  Food Corporation of India (FCI) is responsible for procuring and stocking food grains, while distribution is ensured by the public distribution system (PDS). The FCI procures food grains from the farmers at the government announced minimum support price (MSP) and then is given to the poor at subsidized prices.

10. What is the impact of globalization on agriculture?

Ans: After 1990, under globalization, the farmers in India have faced new challenges in the international market. Despite being an important producer of rice, cotton, rubber, tea, coffee, jute and spices Indian agricultural products are not able to compete with the developed countries because of the highly subsidized agriculture in those countries. Genetic engineering is recognized as a powerful supplement in inventing new hybrid varieties of seeds that can increase production and make farming more profitable. In fact, organic farming is much in vogue today because it is practiced without factory-made chemicals such as fertilizers and pesticides and promotes organic farming.

Class 10 Geography Chapter 4 Notes: Types of Farming

Primitive Subsistence Farming - In primitive subsistence farming, popularly known as ‘slash and burn’ agriculture, farmers clear a patch of land for producing cereals. When soil fertility decreases, they burn the remains on the land and move to a fresh patch of land. This is known as ‘jhumming’ or ‘jhum cultivation’ in the north-eastern states.

Intensive Subsistence Farming - Intensive Subsistence Farming is practised in highly populated land. It is labour-intensive and requires high biochemical inputs.

Commercial Farming - Commercial Framing uses high yielding varieties of seeds, chemical fertilizers, pesticides and insecticides.

Plantation - Plantation is a large tract of land with high capital investment where a single crop is grown. The produce from a plantation is used mostly in industries.

Cropping Patterns in India

In India, there are three cropping seasons:

Rabi - The sowing season is winter from October to December. The harvesting season is summer from April to June.

Kharif - The sowing season for Kharif crop is the rainy season between April and May. The harvesting season is September and October.

Zaid - Zaid crops are grown in a short season between the Rabi and the Kharif season during March and July.

Major Crops Grown in India

The different food and non-food crops grown in India are:

Rice: The cultivation of rice, a Kharif crop requires high temperatures and high humidity. India, after China, is the second-largest rice producer in the world. Rice is cultivated in the Gangetic plains of the north and the northeast fertile plains and the deltaic regions.

Wheat: Wheat requires a cold season for growing and bright sunshine during its ripening. It requires around 50 to 75 cm of annual rainfall. It is a Rabi crop. It is grown mainly in the Ganga-Satluj plains of the north-west and the black soil region of the Deccan in India.

Maize: Maize is a Kharif crop grown mainly in the states of Bihar, Uttar Pradesh, Telangana, Andhra Pradesh, Madhya Pradesh and Karnataka. It is a Kharif crop used as both food and fodder. Alluvial soil is best suited for its growth.

Pulses: Pulses form the major source of protein in a vegetarian diet. India stands as the largest producer and consumer of pulses in the world. They are grown in rotation with other crops as they are leguminous and restore the fertility of the soil.

Millets: They are coarse grains with high nutritional value:

Jowar: Jowar is mostly a rain-fed crop. It is the third most important crop in the country after paddy and wheat with respect to area coverage and production.

Bajra: Bajra is suited to grow on black soil.

Ragi: Ragi is a dry region crop. It grows well on almost all kinds of soil including red, sandy black, loamy, and shallow black soils.

Sugarcane: India stands second in producing sugarcane after Brazil. It requires enormous manual labour as sowing to harvesting has to be done manually. It can be grown in any kind of soil.

Oil Seeds: Cultivation of oil seeds cover 12% of India’s agricultural land. The primary oilseeds are mustard, groundnut, linseed, coconut, etc.

Tea: Tea was introduced by the British in India. It is a labour-intensive industry located in the states of Assam, Darjeeling, Kerala, and Tamil Nadu. Tea cultivation requires extremely well-drained soil, rich in organic matter.

Coffee: The variety of coffee produced in India is Yemen and is in great demand all over the world.

Horticulture Crops: The country produces both tropic and temperate fruits. The major horticulture crops grown in the nation are mangoes, brinjal, potato, onion, cabbage, apples, guava, apricots and walnuts.

Rubber: It is an equatorial crop with a requirement of more than 200 cm of rainfall. It is grown in Kerala, Tamil Nadu, Andaman and Nicobar Islands and on the Garo hills.

Fibres: Cotton, jute and hemp are fibres grown on soil and silk is reared from cocoons of the silkworm. Cotton is a Kharif crop well suited to growing on black soil. Jute grows well in well-drained alluvial soil.

Technological and Institutional Reforms in Agriculture

Agriculture employs more than 60% of the country's workforce. Some adjustments are required to improve this industry. The Green Revolution and the White Revolution were two such reforms implemented to improve the country's agricultural sector. To assist farmers, schemes such as the Kisan Credit Card and the Personal Accident Assurance Scheme were created.

What does the Report Say?

According to reports of 2010-11, 52% of the workforce of the country is employed by agriculture. The Indian Council of Agricultural Research (ICAR) and other agricultural institutions, horticulture development centres, meteorological departments, animal breeding centres, veterinary institutions were introduced aiming to improve agricultural conditions in the country.

Farming is an age-old economic activity that is practised throughout the country. Farming practices have evolved throughout time. Newer approaches for agricultural improvement have been created over time. You may learn more about agriculture by downloading the Agriculture PDF Notes. Agriculture Class 10 PDF can also help you study for your exams.

While preparing for the board exam, students look for last-minute quick revision notes of Geography to revise the complete syllabus quickly. We provide you with the chapter-wise revision notes for all the chapters of Geography, created by our subject-matter experts. These notes will not only help you in quick revision but also understand to the concepts in a concise manner. 

Along with this, students can also view additional study materials provided by Vedantu, for Class 10 Geography

CBSE Class 10 Geography Notes 2024

NCERT Class 10 Geography Chapterwise Solutions

Chapter-1 Resources and Development

Chapter-2 Forest and Wildlife Resources

Chapter-3 Water Resources

Chapter-5 Minerals and Energy Resources

Chapter-6 Manufacturing Industries

Chapter-7 Lifelines of National Economy

Conclusion 

The study of agriculture in Class 10 CBSE Geography provides students with a comprehensive understanding of the agricultural practices and their significance in our society. This chapter delves into the various aspects of agriculture, including types of farming, cropping patterns, agricultural resources, and challenges faced by farmers.

The CBSE Geography textbook provides NCERT Solutions that aid students in grasping the concepts, answering questions, and preparing for examinations. These solutions offer explanations, examples, and case studies that enhance comprehension and analytical skills. In the study of agriculture in Class 10 CBSE Geography plays a crucial role in developing students' knowledge and understanding of the agricultural sector.

arrow-right

FAQs on Agriculture Class 10 Notes CBSE Geography Chapter 4 (Free PDF Download)

1. What is Slash and Burn Agriculture?

Slash and burn is a primitive kind of agricultural technique where the land is cleared for growing crops by cutting down the forest. After the soil loses productivity the land is abandoned and the crop remains burnt. People move to a fresh patch of land and repeat the process. To know more about different types of agricultural techniques download the agriculture class 10 notes.

2. What is the Importance of Agriculture Class 10?

Agriculture employs nearly 60% of India’s population. It is the activity which provides basic food to everybody. It also provides the raw material for several industries. All these reasons make agriculture extremely important. Chapter 4 Geography Class 10 notes will enlighten about agriculture.

3. How should I prepare for Geography Class 10 Chapter 4?

In order to start, first plan an organized routine. Students are advised to allot equal time to each and every topic and complete the day to day tasks regularly. In order to get a better understanding of the subjects, students should refer to the NCERT notes for Class 10 Geography provided free of cost by Vedantu. These notes will prove extremely beneficial for the student in retaining the important concepts and topics, which will thereby help them attain better marks in the examination. 

4. What are the major types of farming?

Primitive Subsistence Farming: This farming is worked on small patches using primitive tools such as digging sticks, family labour etc. Monsoon and the naturally fertile soil are the important aspects of this type of farming. 

Intensive Subsistence Farming: This kind of farming is practised in areas with high populations. Because of this it demands huge labour and uses biochemicals and irrigation to yield good produce. 

Commercial Farming: This kind of farming uses and utilizes modern substances, for instance, fertilizers, HYV, pesticides etc so that the production is good. 

5. How has agriculture contributed to the national economy?

Agriculture has always been the most practised sector in the land of India, therefore, it plays a very prominent role in the Indian Economy.  Most of the households in India depend on agriculture for sustenance. It contributes almost 17% to the total GDP and provides employment to more than 60% of the population. It is important for the student to study this chapter thoroughly to get an idea about the role that agriculture plays in India. 

6. Is Class 10 Geography Chapter 4 easy and scoring?

Chapter 4 is an important subject from an exam point of view. Nevertheless, Social Studies is a subject where the student can easily score high marks just by having a good hold on the basic concepts and ideas. But in order to do that, it is evident that the students need to go through the texts thoroughly and read the chapters line by line, marking and highlighting the important portions and topics. It is also advised that students follow and practice the exercises that the NCERT solutions provided by Vedantu, as that will help them clear all their doubts and queries. All the study material is available on the vedantu app.

7. How does Globalization have an impact on agriculture?

Globalisation, which has existed since colonialism, has always had an influence on agriculture. South Indians were pushed to cultivate spices throughout the European era because it was the principal item of export, and this practise continues to this day. Cotton was also a major raw material shipped to the United Kingdom and subsequently turned into a finished product. But, as a result of globalisation, Indians have confronted new obstacles. This is because, despite being the largest exporter of cotton, tea, rice, and other commodities, it is insufficient to compete with the developed countries. To attain balance, the government must take appropriate efforts to assist small and disadvantaged farmers.

  • Class 6 Maths
  • Class 6 Science
  • Class 6 Social Science
  • Class 6 English
  • Class 7 Maths
  • Class 7 Science
  • Class 7 Social Science
  • Class 7 English
  • Class 8 Maths
  • Class 8 Science
  • Class 8 Social Science
  • Class 8 English
  • Class 9 Maths
  • Class 9 Science
  • Class 9 Social Science
  • Class 9 English
  • Class 10 Maths
  • Class 10 Science
  • Class 10 Social Science
  • Class 10 English
  • Class 11 Maths
  • Class 11 Computer Science (Python)
  • Class 11 English
  • Class 12 Maths
  • Class 12 English
  • Class 12 Economics
  • Class 12 Accountancy
  • Class 12 Physics
  • Class 12 Chemistry
  • Class 12 Biology
  • Class 12 Computer Science (Python)
  • Class 12 Physical Education
  • GST and Accounting Course
  • Excel Course
  • Tally Course
  • Finance and CMA Data Course
  • Payroll Course

Interesting

  • Learn English
  • Learn Excel
  • Learn Tally
  • Learn GST (Goods and Services Tax)
  • Learn Accounting and Finance
  • GST Tax Invoice Format
  • Accounts Tax Practical
  • Tally Ledger List
  • GSTR 2A - JSON to Excel

Are you in school ? Do you love Teachoo?

We would love to talk to you! Please fill this form so that we can contact you

Case Based Questions

  • MCQ Questions (1 Mark)
  • Assertion Reasoning
  • Picture Based Questions (MCQ)
  • True or False
  • Match the following
  • Fill in the blanks (MCQ)
  • Map Based Questions
  • Past Year Questions - 2 Marks
  • Past Year Questions - 3 Marks
  • Past Year Questions - 5 Marks
  • Teachoo Questions

Question 1 - Case Based Questions - Chapter 4 Class 10 Geography - Agriculture - Geography

Last updated at April 16, 2024 by Teachoo

Read the extract given below and answer the questions:

Wheat This is the second most important cereal crop. It is the main food crop, in north and north-western part of the country. This rabi crop requires a cool growing season and a bright sunshine at the time of ripening. It requires 50 to 75 cm of annual rainfall evenly-distributed over the growing season. There are two important wheat-growing zones in the country – the Ganga-Satluj plains in the northwest and black soil region of the Deccan. The major wheat-producing states are Punjab, Haryana, Uttar Pradesh, Bihar, Rajasthan and parts of Madhya Pradesh. Rice It is the staple food crop of a majority of the people in India. Our country is the second largest producer of rice in the world after China. It is a kharif crop which requires high temperature, (above 25°C) and high humidity with annual rainfall above 100 cm. In the areas of less rainfall, it grows with the help of irrigation.

Question (i)

How much annual rainfall is required for wheat.

(A) 50 to 75 cm

(B) below 50 cm

(C) Above 100 cm

(D) 75 to 150 cm

    From the paragraph 

It is the main food crop, in north and north-western  part of the country. This rabi crop requires a cool  growing season and a bright sunshine at the time  of ripening. It requires 50 to 75 cm of annual rainfall  evenly-distributed over the growing season.

So the correct answer is (A) :  50 to 75 cm

Question (ii)

What is climatic conditions required for rice.

(A)High rainfall

(B) High temperature

(C) High humidity

(D) All of the above

From the paragraph

It is a kharif crop that requires high temperature, (above 25°C)  and high humidity with annual rainfall above 100  cm . In the areas of less rainfall, it grows with the  help of irrigation. 

So the correct answer is (D): All of the above 

Question (iii)

Identify the wheat growing zones in india..

(A)The Ganga-Satluj plains in the north-west

(B) Black soil region of the Deccan plateau

(C) Both 'a' and 'b'

(D)None of the above 

There are two important wheat-growing zones in the  country – the Ganga-Satluj plains in the northwest and black soil region of the Deccan . The major  wheat-producing states are Punjab, Haryana, Uttar  Pradesh, Bihar, Rajasthan and parts of Madhya  Pradesh. 

So the correct answer is(C):  Both 'a' and 'b' 

Question (iv)

_________ is the largest producer of rice..

(D) Australia 

From the paragraph 

It is the staple food crop of a majority of the people in India . Our country is the second largest producer  of rice in the world after China . It is a kharif crop  which requires high temperature, (above 25°C)  and high humidity with annual rainfall above 100  cm. In the areas of less rainfall, it grows with the help of irrigation.

So the correct answer is (C) : China

Question (v)

Rice is a _______ crop..

(B) plantation

(C) horticulture

(D) Both B & C 

It is the staple food crop of a majority of the people in India . Our country is the second largest producer of rice in the world after China.

So the correct answer is(A) : Food 

Question (vi)

Identify major difference between wheat crop and rice crop..

(A) Wheat is grown mostly in south India; Rice is grown mostly in north India.

(B) Rice is a Kharif crop; wheat is a rabi crop.

(C) Rice is called grain crop, wheat is called cereal crop.

(D) Both A & C 

 It is the main food crop, in north and north-western part of the country. T his rabi crop requires a cool growing season and bright sunshine at the time of ripening.

Our country is the second largest producer of rice in the world after China. It is a kharif crop  which requires high temperature, (above 25°C)  and high humidity with annual rainfall above 100  cm.

So the correct answer is (B) :  Rice is a Kharif crop; wheat is a rabi crop

Davneet Singh's photo - Co-founder, Teachoo

Davneet Singh

Davneet Singh has done his B.Tech from Indian Institute of Technology, Kanpur. He has been teaching from the past 14 years. He provides courses for Maths, Science, Social Science, Physics, Chemistry, Computer Science at Teachoo.

Hi, it looks like you're using AdBlock :(

Please login to view more pages. it's free :), solve all your doubts with teachoo black.

case study of agriculture class 10

Agriculture Class 10 Important Questions with Answers

Class 10 Geography Chapter 4 Agriculture Important Questions and answers cover these topics and help students to understand the concepts better. Students can solve these for practice. They may come across some of these questions in the board exam.

Students can clear their doubts from the chapter by solving these CBSE Class 10 Geography Important Questions and prepare well for the board exams. The links to download the PDF version of these questions are given in a link in this article.

Class 10 Geography Chapter 4 Agriculture Important Questions

1. Write four points to stress the importance of agriculture in India. (2013) Or, Why is India called an agriculturally important country? Write four supportive arguments. Or, ‘Agriculture has been the backbone of the Indian Economy’ Explain the statement by giving reason. (2017 D)

Answer: India is an agricultural country because of the following reasons:

  • Two-third of its population is engaged in agricultural activities which provide livelihood.
  • Agriculture is a primary activity and produces most of the food and foodgrains that we consume.
  • It produces raw materials for our various industries, e.g., cotton textile and sugar industry.
  • Some agricultural products, like tea, coffee and spices, are exported and earn foreign exchange.
  • The share of agriculture in providing employment and livelihood to the population continued to be as high as 63% in 2001.

2. Why do farming practices vary in different regions? Give three major reasons.

Answer: Three major reasons are:

  • Physical environment, i.e., relief, soil and climate.
  • Technological know-how.
  • Socio-cultural practices.

3. Name some industries based on agricultural raw material.

Answer: Cotton Textile Industry and Sugar Industry.

4. What factors does primitive subsistence farming depend on?

Answer: Factors:

  • Natural fertility of the soil
  • Suitability of other environmental conditions to the crops grown.

5. Write main characteristics of ‘slash and burn’ agriculture. Or, Write main characteristics of Primitive Subsistence Farming.

  • This type of farming is practised in few pockets of India on small patches of land using primitive tools and family/community labour.
  • Farmers clear a patch of land and produce cereals and other food crops to sustain their families.
  • When the soil fertility decreases, the farmers shift and clear a fresh patch of land for cultivation. This type of shifting allows nature to replenish the fertility of the soil through natural processes.
  • Land productivity is low as the farmer does not use fertilisers or other modem inputs.

6. Name some primitive tools used in ‘slash and bum’ agriculture.

Answer: Tools like hoe, dao and digging sticks.

7. Compare ‘intensive subsistence farming’ with that of ‘commercial farming’ practiced in India.

(i) In this type of farming, production is mainly for self consumption.(i) In this, crops are mainly grown for commercial purposes.
(ii) It is practised in areas of high population pressure on land.(ii) It is practiced on large pieces of land on scientific and commercial lines.
(iii) It is labour intensive farming.(iii) In this type of agriculture, machines and modem technology are used.
(iv) In this, high doses of biochemical inputs and irrigation are used for obtaining higher production.(iv) There is higher use of modern agricultural inputs, for example, High Yielding Variety (HYV) seeds, fertilisers, irrigation, etc. are used to obtain higher yields and production.
(v) Farmers take maximum output from the limited land by raising 2-3 crops in a year from the same land, i.e., multiple cropping is practised.(v) The degree of commercialization varies from one region to another. Rice is a commercial crop in Punjab, while in Orissa it is subsistence crop. For example, Plantation agriculture.

8. Write the main characteristics of intensive subsistence farming.

  • It is practised in areas of high population pressure on land.
  • It is labour intensive farming.
  • Yield per hectare is high because high doses of biochemical inputs and irrigation are used.
  • The size of the land-holdings is small and uneconomical.
  • Farmers take maximum output from the limited land by raising 2-3 crops in a year from the same land, i.e., multiple cropping is practised.

9. What are the major differences between primitive subsistence farming and commercial farming? (2013) Answer:

1. In this mainly cereals and other food crops are grown by farmers to sustain themselves.1. In this, crops are mainly grown for commercial purposes.
2. It is generally done on small land holdings which are economically not viable.2. It is practiced on large pieces of land on scientific and commercial lines.
3. Primitive tools and animals are used for carrying out agricultural activities.3. In this type of agriculture, machines and modern technology are used.
4. In this, modern agricultural inputs, e.g., fertilisers and irrigation are not widely used.4. There is higher use of modern agricultural in­puts, e.g., HYV seeds, fertilisers, irrigation, etc. are used to obtain higher yields and production.
5. Subsistence Agriculture is called Jhumming in Assam, Mizoram; Kuruwa in Jharkhand. E.g., Slash and Burn.5. The degree of commercialization varies from one region to another. Rice is a commercial crop in Punjab, while in Orissa it is subsistence crop. E.g., Plantation agriculture.

10. What is the most important characteristic of commercial farming?

Answer: The main characteristic of commercial farming is the use of higher doses of modern inputs, e.g., high yielding varieties seeds (HYVs), chemical fertilizers and pesticides to increase productivity. The degree of commercialisation of agriculture varies from one region to another.

11. Give an example of a crop which is commercial in one region and provides subsistence in another. (2012)

Answer: Rice is a commercial crop in Haryana and Punjab, while in Odisha it is a subsistence crop.

12. Name one type of agriculture which falls in the category of commercial agriculture. Write the main characteristics of this type of agriculture. (2012) Or Write six characteristics of plantation agriculture.

Answer: Plantation agriculture is a type of commercial farming. Characteristics of plantation agriculture:

  • A single type of crop is grown on a large area.
  • Plantation is carried out on large estates using lot of capital intensive units.
  • Lot of migrant labourers work on these estates.
  • The plantation has an interface of agriculture and industry. All the produce is used as raw material in the respective industries.
  • The production is mainly for the market, i.e., commercial agriculture.
  • A well developed network of transport and communication connecting the plantation areas, processing industries and markets plays an important role in the development of plantations.

13. Name one horticultural plantation crop and two beverage plantation crops and two States each which specialise in their production respectively.

Answer: Horticultural plantation crop is apples, mainly grown in Himachal Pradesh and Jammu & Kashmir. Beverage crops:

  • Tea which is mainly a plantation crop in Assam and North-West Bengal.
  • Coffee which is grown in Karnataka.

14. Name some important plantation crops.

Answer: Tea, coffee, rubber, sugarcane and banana are examples of some important plantation crops.

15. Name three cropping seasons of India. Write their sowing and harvesting time and major crops of each season.

Answer: Rabi, Kharif and Zaid are the three cropping seasons.

Rabi crops (Winter crops): These are sown in winter from October to December. Harvested in summer from April to June. Important crops are wheat, barley, mustard, peas, gram etc.

Kharif crops (Crops of the rainy season): These are sown with the onset of monsoon (June-July) and are harvested in September-October. Important crops are rice, maize, millets, jowar, bajra, tur (arhar), moong, urad, cotton, jute, groundnut and soyabean.

Zaid season: It is a short cropping season during summer months (mainly between March-April and June-July in different parts of the country). Important crops are watermelon, musk melon, cucumber and fodder crops, etc.

16. Where are rabi crops mainly grown? Describe the climatic conditions required for their growth.

Answer: Rabi crops such as wheat and other crops are grown mainly in states from the north and north-western parts such as Punjab, Haryana, Himachal Pradesh, Jammu & Kashmir, Uttaranchal and Uttar Pradesh. Climatic conditions: Availability of precipitation during winter months due to western temperate cyclones help in successful growth. The green revolution has also been an important factor in the growth of rabi crops.

17. Name some important rice growing regions of India.

Answer: Assam, West Bengal, Coastal regions of Orissa, Andhra Pradesh, Tamil Nadu, Kerala and Maharashtra etc.

18. Name three states which raise three paddy crops in a year. Name these crops also.

Answer: Assam, West Bengal and Orissa grow three crops in a year. The crops are Aus, Aman and Boro.

19. (a) Which is the most important food crop of India? What is its position in world production? Or, Name the food crop of Kharif season. What is India’s position in the world with regard to its production? (b) Write the rainfall and temperature requirements for the growth of this crop. (c) Name four major regions of rice cultivation. (d) Name four states which produce irrigated rice.

Answer: (a) Rice is the most important food crop (Kharif crop) of India. India holds second position in rice production after China.

(b) For rice cultivation, high temperature of 25°C and above and high humidity with annual rainfall of 100 cms is required.

(c) Four major regions of rice cultivation are:

  • Plains of North India
  • Plains of North-Eastern India
  • Coastal areas
  • Deltaic regions.

(d) Irrigated rice is produced in Punjab, Haryana, Western Uttar Pradesh and Rajasthan. Development of dense network of canals and tubewells has made it possible to grow rice in these states which receive less rainfall.

20. (a) Which is the second most important cereal crop of India? Write the geographical conditions required for its growth. (b) Name two major zones of wheat growth and name six major states of wheat production.

Answer: (a) Wheat is the second most important cereal crop of India. It is the main food crop. Geographical conditions:

  • Cool and moist growing season.
  • Bright sunshine at the time of ripening.
  • Rainfall: 50 to 75 cm evenly distributed over the growing season.
  • Loamy soil is best for its growth.

(b) Two important wheat growing zones in India are:

  • The Ganga-Sutlej plains in the North-West and
  • Black soil region of the Deccan.

The major wheat producing states are: Punjab, Haryana, Uttar Pradesh, Bihar, Rajasthan and Madhya Pradesh.

21. Which is the third most important food crop of India? Name the cropping season in which it is grown. Name four leading states of its production.

Answer: Jowar is the third most important food crop with respect to area and production. It is mainly the crop of Kharif season. It is a rainfed crop grown in moist areas. Leading states of production are: Maharashtra, Karnataka, Andhra Pradesh and Madhya Pradesh.

22. What are coarse grains? Why are they important in India? Name the crops which are included in this category and name three leading states producing each of these crops.

Answer: Millets are called coarse grains. They are important because they have high nutritional value and make an important part of the diet for poor people. Most important millets are as follows:

  • Ragi —Leading producer is Karnataka, followed by Tamil Nadu. Himachal Pradesh, Jharkhand, Uttaranchal, Sikkim etc are other important regions.
  • Jowar—Maharashtra is the leading producer followed by Karnataka, Andhra Pradesh, Madhya Pradesh.
  • Bajra—It grows well on sandy soils and shallow black soils. Rajasthan is the largest producer followed by Uttar Pradesh, Maharashtra, Gujarat and Haryana.

23. Write two geographical conditions required for the growth of Ragi and write its nutritional value.

Answer: Geographical conditions:

  • Ragi grows well in dry regions.
  • It grows well on red, black, sandy, loamy and shallow black soils.

Nutritional value: Ragi is rich in iron, calcium, other micro-nutrients and roughage.

24. Mention two geographical conditions required for the growth of Maize crop in India. Describe three factors which have contributed to increase maize production. Write four major maize producing states. (2012)

Answer: Geographical conditions required for the growth of maize crop in India:

  • It is a kharif crop which requires temperature between 21° C to 27° C. It requires moderate rainfall between 50-100 cm.
  • use of modern inputs such as HYV seeds;
  • use of fertilisers; and
  • use of irrigation facilities.
  • major maize producing state: Karnataka, Uttar Pradesh, Bihar, Andhra Pradesh and Madhya Pradesh.

25. (a) Name three pulses each of Rabi and Kharif season. Write their importance for human beings and for agriculture. (b) What is India’s position in the world with regard to the production of pulses? Name five leading states producing pulses. (2015)

Answer: (a) Pulses of Rabi season: Tur (arhar), urad, moong. Pulses of Kharif season: Masur, peas, gram. Importance of pulses:

  • For agriculture. Being leguminous crops, they help in restoring soil fertility by utilising nitrogen from the air (nitrogen fixation). Therefore, these are mostly grown in rotation with other crops.
  • They need less moisture and survive even in dry conditions.

(b) India is the largest producer of pulses in the world. Major pulse producing states are: Madhya Pradesh, Uttar Pradesh, Rajasthan, Maharashtra and Karnataka.

26. (a) What is India’s position in the world regarding sugarcane production? Write the geographical conditions required for its growth. (b) Write the major states that produce sugarcane. (c) Name four products obtained from sugarcane.

Answer: (a) India is the second largest producer of sugarcane after Brazil. Geographical conditions: It is a tropical as well as subtropical crop.

  • It grows well in hot and humid climate.
  • Temperature: 21°C to 27°C.
  • Annual rainfall between 75 cm and 100 cms. Irrigation is required in the regions of low rainfall.
  • It can be grown on a variety of soils.
  • It needs manual labour from sowing to harvesting.

(b) Six major states producing sugarcane are: Uttar Pradesh, Maharashtra, Karnataka, Tamil Nadu, Andhra Pradesh, Bihar, Punjab and Haryana.

(c) Sugarcane is the main source of sugar, gur (jaggery), khandsari and molasses.

27. (a) What percentage of the total cropped area of India is under oilseed production? What is India’s position in the world with regard to oilseed production? (b) Name six oilseeds produced in India. What are their main uses? (c) Which oilseed constitutes 50% of the oilseeds produced in the country? Name three major states producing this oilseed. (d) Name three oilseeds of Kharif season and three of Rabi season.

Answer: (a) India is the largest producer of oilseeds in the world. 12% of the total cropped area is under oilseed production

(b) Six major oilseeds produced in India are: Groundnut, mustard, coconut, sesamum (til), soyabean, castor-seeds, linseed, sunflower and cotton-seeds. Most of these oilseeds are edible and are used as cooking mediums. Some are also used as raw material in the production of soap, cosmetics and ointments.

(c) Groundnut is a Kharif crop and constitutes 50% share in the total oilseed production. Andhra Pradesh, Tamil Nadu, Karnataka, Gujarat and Maharashtra are important producers of groundnut.

Oilseeds of Kharif seasonOilseeds of Rabi season
GroundnutMustard
Sesamum in North IndiaLinseed
Castor-seedSesamum in South India

28. Write the geographical and labour conditions required for the growth of tea.

  • It grows well in tropical and subtropical climates.
  • It requires deep, fertile, well drained soil, rich in humus and organic matter.
  • It requires warm and moist frost free climate round the year.
  • Frequent showers evenly distributed through the year ensure continuous growth of tender leaves.
  • Tea is a labour intensive industry. It requires abundant, cheap and skilled labour.
  • It is processed within tea gardens to restore its freshness.

29. What is India’s position in the world regarding tea production? Name three states each which produce tea in: (i) North-Eastern India (ii) Himalayan states/parts of the states (iii) Peninsular States.

Answer: India is the leading producer and exporter of tea in the world. Three states producing tea in each of the following:

North-Eastern StatesHimalayan Regions/StatesPeninsular States
1. Assam1. Hills of West Bengal1. Tamil Nadu
2. Meghalaya(Darjeeling & Jalpaiguri Districts)2. Kerala
3. Tripura2. Himachal Pradesh3. Andhra Pradesh
3. Uttaranchal

30. Which is the most important beverage crop of Southern India and what is its percentage share in the world? Where did its cultivation initially start in India? Name three major states of its production.

Answer: Coffee is the most important beverage crop of Southern India. India produces about 4% of the world’s coffee production. Three major states which produce coffee are: Karnataka, Kerala and Tamil Nadu, mainly in Nilgiri Hills. Initially its cultivation started in Baba Budan Hills.

31. Name a good variety of coffee which is produced in India and name the country from where it was initially brought. Why Indian coffee has great international demand?

Answer: Arabica, the good variety of coffee was originally brought from Yemen. Indian coffee is in great demand because this coffee is of a very good quality.

32. What is horticulture? (2012)

Answer: Cultivation of fruits, vegetables and flowers is called horticulture.

33. What is India’s position in the world in the production of fruits and vegetables? Name the different Indian fruits which are in great demand world over and also name the states where each is produced.

  • India is the largest producer of fruits and vegetables in the world.
  • Mangoes grow in Maharashtra, Andhra Pradesh, Uttar Pradesh and West Bengal.
  • Bananas grow well in Kerala, Mizoram, Maharashtra and Tamil Nadu.
  • Oranges grow well in Nagpur and Cherrapunjee (Meghalaya).
  • Lichi and Guavas grow in Uttar Pradesh and Bihar.
  • Grapes grow well in Andhra Pradesh and Maharashtra.
  • Pineapple grows well in Meghalaya.
  • Apples, Pears, Apricots and Walnuts grow well in Jammu and Kashmir and Himachal Pradesh.

34. Name four temperate fruits which are produced in India and name the states which grow them in abundance.

Answer: Four temperate crops are: apples, pears, apricots and walnuts. Himachal Pradesh and Jammu & Kashmir are the leading producers of these temperate fruits.

35. Describe India’s position in vegetable production.

  • India produces about 13% of the world’s vegetables.
  • It stands first in the production of peas and cauliflower.
  • It stands second in the production of onions, cabbage, tomatoes and brinjal and stands fourth in the production of potatoes.

36. Name three non-food crops and write three major states of their production respectively.

Answer: The three non-food crops are: rubber, cotton and jute. Rubber is produced in Kerala, Tamil Nadu and Karnataka, Andaman and Nicobar islands and Garo hills of Meghalaya. Cotton is mainly produced in Maharashtra, Gujarat and Madhya Pradesh. Jute is mainly grown in West Bengal, Bihar and Assam.

37. Write the geographical conditions required for the growth of rubber. Name the item which consumes maximum share of rubber for its manufacture.

Answer: Conditions:

  • It requires hot and humid climate.
  • Rainfall – 200 cms.
  • Temperature – above 25°C.

Maximum rubber is consumed in the manufacture of auto tyres and tubes and cycle tyres and tubes. They together consume about 57.8% of the total rubber production.

38. Name four major fibre crops grown in India. What is ‘sericulture’?

Answer: Cotton, jute, hemp and natural silk are the four major fibre crops grown in India. The first three are derived directly from the crops grown in the soil, but silk is obtained from the cocoons of silk worms fed on mulberry leaves. Sericulture: Rearing of silk worms for the production of silk fibre is known as sericulture.

39. Give an account of fibre crop which is mainly grown in Deccan Plateau region under the following heads: (i) Its position in the world production (ii) geographical conditions and (iii) major states of production.

Answer: Cotton is the fibre crop which is mainly grown in the black soil of the Deccan Plateau region. (i) Position. India is the 3rd largest producer of cotton in the world. (ii) Geographical conditions. Cotton requires:

  • high temperature.
  • light rainfall or irrigation.
  • 210 frost-free days.
  • bright sunshine for its growth.
  • black cotton soil which is very good for its growth.
  • It is a Kharif crop and requires 6-8 months to mature.

(iii) Major cotton producing states are. Maharashtra, Gujarat, Madhya Pradesh, Karnataka, Andhra Pradesh, Tamil Nadu, Punjab, Haryana and Uttar Pradesh.

40. Which fibre crop is called as the ‘Golden fibre’? Which is the most important region of its growth and why? Give one major reason why it is losing the market now.

Answer: Jute is known as the golden fibre. Jute is mainly grown in West Bengal, especially in the Hooghly Basin because there the geographical conditions favour its growth. These conditions are:

  • High temperature required during the time of growth.
  • Jute grows well on well-drained fertile soils in the flood plains where soils are renewed every year. Due to its high cost, it is losing market to synthetic fibres and other packing materials particularly to nylon.

41. Name the major jute producing states and list some items or products made out of jute.

Answer: Jute producing states are: West Bengal, Bihar, Assam, Orissa and Meghalaya. Jute products are: gunny bags, ropes, mats, carpets, yarns and other ornamental artefacts.

42. Give the main reasons which have necessitated agricultural reforms.

Answer: Reforms in agriculture are necessary because of the following reasons:

  • Sustained uses of land without compatible techno-institutional changes have hindered the pace of agricultural development.
  • Most of the farmers still depend on monsoons because large parts of the country still do not have irrigation facilities.
  • Farmers still depend on natural fertility in order to carry on their agriculture, i.e., they lack material resources, e.g., fertilizers, etc.
  • Ours is an agricultural economy and about 63% of people depend on agriculture for employment and livelihood, therefore reforms have to be implemented.
  • For raising the agricultural production and productivity levels to produce sufficient food for the growing population.
  • To overcome environmental, economic and social constraints, agricultural reforms have to be seriously implemented.
  • The declining share in GDP is a matter of serious concern because decline and stagnation in agriculture will lead to decline in other spheres of economy.

43. What are ‘Institutional Reforms’? Enlist various institutional reforms taken by the Indian Government to bring about improvements in agriculture. (2015)

Answer: Steps taken by the government to bring about improvements in agriculture are termed as ‘Institutional Reforms’.

Some steps are:

  • Collectivisation and consolidation of land holdings to make them economically viable.
  • The green revolution based on the use of package technology and the White Revolution to increase milk production are important strategies which were initiated to improve agriculture.
  • Cooperation with farmers and Abolition of Zamindari system.
  • Provision of crop insurance to protect the farmers against losses caused by natural calamities, i.e. drought, flood, cyclone, fire and disease.
  • Establishment of ‘Grameen Banks’, Cooperative Societies and Banks for providing loan facilities to the farmers at lower rates of interest.
  • Kissan Credit Card (KCC), Personal Accident Insurance Scheme (PAIS) are some other schemes introduced by the government for the benefit of farmers.
  • Special weather bulletins and agricultural programmes for farmers were introduced on the Radio and TV.
  • Announcement of minimum support price, remunerative and procurement prices for crops to check the exploitation of farmers by speculators and middlemen and removing the elements of uncertainty.

44. Describe some concerted efforts made by the government to modernise Indian agriculture and improve its share in the GDP.

Answer: The following measures have been taken by the government to modernise agriculture and improve its share in the GDP:

  • Establishment of Indian Council of Agricultural Research (ICAR).
  • Setting up of Agricultural Universities.
  • Development of advanced Veterinary Services and Animal Breeding Centres.
  • Development of horticulture.
  • Research and Development in the field of meteorology and weather forecasts, etc. Improving the rural infrastructure is essential for the same.

45. “The contribution of agriculture to national economy is on the decline.” Write five facts to support this statement. Or, What is the contribution of agriculture to national economy, employment and output? Is it declining or encouraging?

Answer: Contribution of agriculture is showing a declining trend:

  • The share of agriculture in GDP has shown a declining trend since 1951.
  • Yet its share in providing employment and livelihood to the population continued to be as high as 63% in 2001.
  • Although GDP growth rate of the country is increasing, it is not generating sufficient employment opportunities in the country, especially in the field of agriculture.
  • Farmers are dragging away their investment from agriculture because they have to face big challenges from international competition. This has resulted in further downfall in employment in agriculture.
  • The growth rate in agriculture is decelerating which is an alarming situation.

46. What are the challenges being faced by Indian farmers? What has this resulted in?

Answer: Challenges faced by Indian farmers:

  • Reduction in public investment by government in the agricultural sector particularly in areas of irrigation, power, rural roads, market and mechanisation.
  • Subsidy on fertilizers has decreased leading to increase in the cost of production.
  • Reduction in import duties on agricultural products has proved detrimental to agriculture in the country. All these factors have led to stiff international competition. Farmers are thus withdrawing their investment from agriculture causing a downfall in agricultural employment.

47. Define each of the following: (a) Green Revolution (b) White Revolution (c) Yellow Revolution (d) Blue Revolution (e) Gene Revolution.

Answer: (a) Green Revolution. Agricultural Revolution which resulted in increased production of foodgrains because of the use of HYV (High Yielding Varieties) seeds, fertilizers, proper irrigation and other modem inputs. (b) White Revolution. Revolutionary increase in the production of milk which was spearheaded by Operation Flood. (c) Yellow Revolution. Increase in the output of oilseeds is termed as Yellow Revolution. (d) Blue Revolution refers to increased output of fish and fish products. (e) Gene Revolution refers to the development of genetically modified seeds for increasing the yield per hectare. These seeds are environmentally sustainable.

48. Write four drawbacks of green revolution.

  • Due to overuse of chemicals land degradation has taken place.
  • Excessive irrigation caused drying of aquifers.
  • It became a cause for vanishing biodiversity.
  • It has widened the gap between poor and rich farmers because only agriculturally rich areas have benefitted from it and not the small farmers.

49. (a) Give four reasons why Indian farmers should switch over from cereals to high value crops’ cultivation. (b) What will be the implication of this change? Name three countries which have successfully done this.

Answer: (a) Indian farmers should switch from cereals to high value crops because of the following reasons:

  • It will improve their income.
  • It will reduce environmental degradation at the same time.
  • Fruits, medicinal herbs, bio-diesel crops, flowers and vegetables need much less irrigation than rice or sugarcane.
  • India’s wide variety of climates can be harnessed to grow a wide range of high value crops.

(b) Its implications are:

  • India has to import food.
  • If we import food while exporting high value crops, our economy will grow.
  • Three countries which have such successful economies are: Chile, Israel and Italy, which export farm products (fruits, wine, olives, speciality seeds) and import food.
  • School Guide
  • Class 10 Syllabus
  • Maths Notes Class 10
  • Science Notes Class 10
  • History Notes Class 10
  • Geography Notes Class 10
  • Political Science Notes Class 10
  • NCERT Soln. Class 10 Maths
  • RD Sharma Soln. Class 10
  • Math Formulas Class 10

CBSE Notes Class 10 Geography Chapter 4 – Agriculture

CBSE Notes Class 10 Geography Chapter 4- Agriculture is about agriculture and the agricultural sector in India. Agriculture is the prime activity of the economy of India and produces raw materials for various types of industries. We will focus on and study the types of farming, cropping patterns, and also the various major crops that are grown in India. We will also learn about the contribution of agriculture to the national economy, to employment, and also about the outputs emerging from agriculture.

For easy accessibility to the students, we have tried to compile CBSE Class 10 Social Science Notes which mostly consist of the four main domains: History, Geography, Political Science, and Economics, in a single source, and also the access to the same is free of cost.

Table of Content

Topics Covered in Chapter 4 Agriculture of Class 10

Highlights of chapter 4- agriculture, cbse notes class 10 geography chapter 4- agriculture, types of farming, cropping pattern, major crops, food crops other than grains, non-food crops, technological and institutional reforms, contribution of agriculture to the national economy, employment, and output, food security, impact of globalization on agriculture.

Agriculture Notes

Agriculture Notes

Below are the important topics discussed under chapter “Agriculture”

Primitive Subsistence Farming

Intensive Subsistence Farming

Commercial Farming

  • Major Crops- Food crops and Non food crops
  • Contribution of Agriculture to National Economy
  • Impact of Globalisation on Agriculture

Board

CBSE

Textbook

Contemporay India- II

Class

10

Subject

Geography (Social Science)

Chapter

4

Chapter Name

Agriculture

Category

Notes

Types of farming and their applicability depend on several factors which include the physical environment, technological progress along with socio-cultural practices. Types of farming vary from subsistence type to commercial types of arming and in different parts of India, depending on various climates and physical factors, different types of farming are practiced.

Also known as slash-and-burn agriculture, primitive subsistence farming is widely practiced in small patches with the help of primitive tools like dao, hoe, and digging sticks. There is a lot of involvement from family and community labor.

  • This type of farming is heavily dependent on nature as the production and output are determined by the monsoon, fertility of soils, and suitability of various environmental factors.
  • The process is simple where a patch of land is selected and cleared. The crops are grown on this land. Once, the fertility of the soil decreases, people shift to a new land and also the old patch to get replenished with the required nutrients so that it can be used again in the next season.
  • No modern equipment or advanced farming techniques are deployed in this type of farming leading to low land productivity. Primitive subsistence farming is also known as jhumming in northeast India.
  • This is a labor-intensive type of farming where high doses of biochemical inputs and irrigation techniques are used to achieve higher production.
  • This farming is common in areas where less land holding is available.
  • Here, high doses of modern inputs like insecticides, fertilizers, pesticides, and high-yielding variety (HYV) seeds are used to achieve higher productivity. Rice is an example of a commercial crop cultivated in states like Haryana.
  • The plantation is also a type of commercial farming where a crop is grown over a large area. Tea, rubber, sugarcane, and coffee are examples of plantation crops in India.
Read in Detail: Types of Farming in India

There are cropping seasons in India.

1. Rabi 2. Kharif 3. Zaid

October to December (Winter) April-May (Beginning of the rainy season) Between the Rabi and Kharif seasons (March to July)
April to June (Summer) September- October
Wheat, gram, barley, mustard, and peas Paddy, jowar, bajra, maize, types of dals- moong, urad, tur, jute, cotton, soybean, groundnut Watermelon, cucumbers, muskmelon, vegetables, fodder crops.
Read in Detail- Cropping Patterns

In India, a number of major crops are grown based on the soil types, climate, and cultivation processes. Major crops in India are as follows:

  • Coffee 
  • It is a staple food crop. India is the second largest producer of rice after China.
  • A type of Kharif crop, rice is grown in regions with high humidity having an annual rainfall of over 100 cm and elevated temperatures of over 25°C
  • It is cultivated in north and northeast India, coastal regions, and the deltaic regions.
  • It is the second most important cereal crop and is the main food crop of north and northwest India.
  • It requires a cool climate season with 50 to 75 cm of annual rainfall to grow and bright sunshine during the time of ripening.
  • The two most wheat-growing regions of India are – the Ganga Satluj plains of the northwest and the black soil regions of the Deccan. Punjab, Haryana, Bihar, Uttar Pradesh, Madhya Pradesh, and Rajasthan are the wheat-producing states of India.
  • Jowar, bajra, and ragi are the most prominent and important millets cultivated in India.
  • Jowar is a rain-fed crop grown in the states of Andhra Pradesh, Madhya Pradesh, Maharashtra, and Karnataka.
  • Bajra is cultivated on sandy soils and shallow black soils. They are mainly produced in Maharashtra, Gujarat, Rajasthan, Uttar Pradesh, and Haryana.
  • Ragi grows best on black, red, sandy, loamy, and shallow black soils. Tamil Nadu, Karnataka, Uttarakhand, Himachal Pradesh, Sikkim, and Arunachal Pradesh are the major ragi-producing states.
  • It is a Kharif crop that grows well in old alluvial soil. It requires a temperature, of 21°C to 27°C to grow.
  • Karnataka, Bihar, Uttar Pradesh, Andhra Pradesh, Telangana, and Madhya Pradesh are the major maize-producing states.
  • India is the largest producer as well as the largest consumer of pulses in the world.
  • Pulses require less moisture and can even survive in dry conditions.
  • They are a major source of protein in a vegetarian diet and are produced in Maharashtra, Karnataka, Madhya Pradesh, Uttar Pradesh, and Rajasthan.
Read in Detail- Major Food Crops in India >
  • India is the second largest producer of sugarcane in the world (after Brazil).
  • It can be classified both as a tropical and subtropical crop.
  • It is cultivated in regions with hot and humid climates with temperatures ranging from 21°C to 27°C and rainfall between 75 cm and 100 cm.,4. Major producers of sugarcane in India are Maharashtra, Karnataka, Andhra Pradesh, Telangana, Uttar Pradesh, Bihar, Punjab, and Haryana.
  • In 2008, India was the second largest producer of groundnut in the world (after China).
  • 12% of the total cropped area of the country is utilized to grow different oil crops. The major oil seeds that are produced in India are groundnut, mustard, coconut, sunflower, cotton seeds, soybean, sesamum (til), castor seeds, and linseed.
  • Oil crops have multiple uses. They can be used as cooking mediums as well as raw materials for the production of soap, ointments, and cosmetics.
  • Groundnut is a labor-intensive crop while linseed and mustard are rabi crops. Sesamum is a kharif crop in North India and a rabi crop in South India.
  • 1. In 2008, India ranked third among the largest tea producers in the world after China and Turkey.
  • 2. A type of plantation agriculture, tea is an important beverage crop that was introduced in India by the British.
  • It is a labor-intensive industry and requires a warm and moist frost-free climate with frequent showers all throughout the year.
  • Major tea-producing states are Assam, West Bengal (hills of Darjeeling and Jalpaiguri districts), Kerela, and Tamil Nadu.
  • India is famous for its good quality coffee all around the world. As per data collected in 2008, India produced around 3.2% of the world’s total coffee production.
  • It is cultivated mainly in the Nilgiri hills in Karnataka, Kerela, and Tamil Nadu.

Horticulture Crops

  • In 2008, India was the 2nd largest producer of fruits and vegetables after China. Around 13% of the world’s vegetables are produced in India.
  • India is home to both tropical and temperate fruits.
  • Banana: Mizoram, Kerela, Tamil Nadu, Maharashtra
  • Mango: Maharashtra, Telangana, Andhra Pradesh, West Bengal
  • Orange: Meghalaya (Cherrapunjee), Maharashtra (Nagpur)
  • Lichi & Guava: Uttar Pradesh and Bihar
  • Apples: Jammu and Kashmir, Himachal Pradesh
  • India is the 4th largest natural rubber producer in the world.
  • Rubber is mainly an equatorial crop, but under certain conditions, it can be grown in sub-tropical and tropical conditions.
  • It requires a humid and moist climate an annual temperature of 25°C and annual rainfall of more than 200 cm.
  • Rubber is grown in Karnataka, Tamil Nadu, Kerala, Meghalaya (Garo Hills), and Andaman and Nicobar Islands.

Fiber crops

Major fiber crops are grown in India: Silk, Cotton, Jute            

  • It is obtained from the cocoons of silkworms that feed on the leaves of mulberry plants. The process of cultivation of silkworms to obtain silk is called sericulture.
  • Assam, Karnataka, and Andhra Pradesh are the largest, of silk in India.
  • In 2008, India was the second largest producer of cotton in the world (after China).
  • A type of Kharif crop, it grows well in the drier parts of black cotton soil prevalent in the Deccan plateau.
  • It requires a climate of elevated temperature, light rainfall, 210 frost-free days, bright sunshine, and proper irrigation. It takes around 6-8 months to mature.
  • Major cotton-producing states are Uttar Pradesh, Haryana, Punjab, Andhra Pradesh, Telangana, Tamil Nadu, Maharashtra, Gujarat, Madhya Pradesh, and Karnataka.
  • Known as golden fiber, it requires an elevated temperature to grow.
  • It grows on the well-drained fertile soils in the flood plains which get replenished and renewed every year.
  • Jute is used to produce different products like gunny bags, yarn, ropes, mats, and other artifacts.
  • West Bengal, Assam, Odisha, Bihar, and Meghalaya are major jute-producing states.
Read in Detail- Non- Food Crops in India

Agriculture is the source of livelihood for more than 60% of India’s population. After achieving independence from the British, several reforms such as consolidation of holdings, cooperation, abolition of zamindari, collectivization, etc. were introduced and prioritized.

  • During the 60s and the 70s, The Green Revolution and the White Revolution (Operation Flood) were adopted to improve the agriculture sector.
  • In the 1980s and the 1990s, a comprehensive land development program that consisted of both technical and institutional reforms was introduced. Some of the major initiatives that were undertaken were the establishment of Grameen banks, providing crop insurance against natural calamities like floods, cyclones, drought, fire, and disease, and giving loans to farmers at lower interest rates through cooperative societies and banks.
  • Schemes like Kisan Credit Card (KCC), and Personal Accident Insurance Scheme (PAIS) were introduced by the Indian government to help the farmers.
  • Government initiatives like minimum support prices, and remunerative and procurement prices for important crops provide protection to farmers from possible exploitations from speculators and middlemen.
Read in Detail- Technological and Institutional Reforms of Agriculture
  • 52% of the total Indian workforce was employed in the farming sector in 2010-11.
  • Major initiatives like the establishment of the Indian Council of Agricultural Research (ICAR), agricultural universities, horticulture development, animal breeding, and veterinary services, development in meteorology, and weather forecasting have all been given priority to improve the agricultural sector.
  • However, even though the growth rate of India’s GDP is increasing, it is not generating sufficient employment opportunities in the country.
Read in Detail- Contribution of Agriculture to National Economy
  • If any part of the population does not have access to food, then it is regarded as a failure of food security. In India, remote areas are highly prone to natural disasters and uncertainty in food supply.
  • The government has designed a national food security system that is composed of two components- buffer stock and public distribution system (PDS)
  • Food Corporation of India (FCI) looks after the procurement and stocking of food grains while PDS is responsible for distribution.
  • The FCI procures food grains from farmers at the minimum support price (MSP) declared by the government. The PDS is responsible for supplying these food grains at subsidized rates in rural and urban areas.
Read in Detail- Agriculture and Food Security
  • The start of globalization in the early 1990s exposed India to numerous challenges. Despite being a major producer of rice, jute, tea, coffee, spices, and other products, India was unable to compete with developed countries due to their highly subsidized agriculture.
  •  Genetic engineering is a powerful supplement that is helping in creating new hybrid varieties of seeds which is helping in increasing production and making farming more profitable.
  • Organic farming is also the new norm that it doesn’t use any factory-made chemicals such as pesticides and fertilizers.
Read in Detail- Impact of Globalisation on Agriculture

FAQs on CBSE Notes Class 10 Geography Chapter 4 – Agriculture

Why is agriculture an important occupation.

Agriculture is considered to be an important occupation as it includes both cultivation of crops and the breeding of livestock and farmers play important role in development of the country.

Which are the best suitable seasons for agriculture in India?

Depending on the monsoon seasons, agriculture in India and monsoon rains play an important role in the growth as well as the cultivation of crops.

Which are the major crops which are grown in India?

Major crops included are rice, wheat, millet, tea, and coffee.

Please Login to comment...

Similar reads.

  • School Geography
  • School Learning
  • Social Science
  • Chapterwise-Notes-Class-10

Improve your Coding Skills with Practice

 alt=

What kind of Experience do you want to share?

  • CBSE Class 10
  • CBSE Class 10 Important Questions
  • CBSE Class 10 Geography Important Questions
  • Geography Chapter 4 Agriculture

CBSE Class 10 Geography Chapter 4 Agriculture Important Questions

Students can learn about concepts such as types of farming, cropping patterns, major crops, and so on from Chapter 4 of CBSE Class 10 Geography. One of the best ways to understand these concepts thoroughly is to solve the CBSE Class 10 Geography Chapter 4 Agriculture Important Questions. It will also give students an idea about the types of questions that are likely to be repeated in the board exams. Practising these questions will help in scoring high marks on the Social Science paper.

These CBSE Class 10 Geography important questions are taken from the textbooks and question papers, thus covering all critical points of the chapter. Students can also find the download link for these questions below.

Download Free CBSE Class 10 Geography Chapter 4 Agriculture Important Questions PDF

Chapter 4 Agriculture Important Questions

1 . Which one of the following is a rabi crop?

(b) Millets

2 . Mention some of the states of India where Intensive Subsistence Farming is practised.

3. Give some examples of crops which may be commercial in one region and may provide subsistence in another region?

4 . Name one important beverage crop and specify the geographical conditions

required for its growth.

5. Enlist the various institutional reform programmes introduced by the government in the interest of farmers.

6 . Describe the impact of globalisation on Indian agriculture.

7 . What are the geographical conditions required for the growth of rice? Explain.

8 . What are the 3 main features of a ‘Rabi crop season?’

9. Define the 3 main features of a ‘Kharif crop season’.

10. Name any two major fibre crops grown in India. Describe the conditions

required to grow these two crops.

11 . Mention any five institutional reform programmes introduced by the government in the interest of the farmers.

12. Explain the climatic conditions, temperature and rainfall required for the production of jowar in India.

13 . What are the four characteristics of Substinence farming?

15. Mention the three characteristics of commercial farming in India.

16 . State the main function of the public distribution system.

17 . Define the climatic conditions required for the growth of a tea plantation. Mention any two major tea-producing states.

18 . Describe Sugar cane crops and write a short note on them.

19. Define commercial farming v/s plantation farming.

20 . Which are the states that lead in the production of millet in India?

21.  Why do farming practices vary in different regions? Give three major reasons.

22.  Name some primitive tools used in ‘slash and bum’ agriculture.

23.  What is horticulture?

We also provide more study materials such as CBSE Question papers , syllabus and NCERT textbooks to prepare for the exams. Stay tuned to BYJU’S for the latest update on CBSE/ICSE/State Board/Competitive Exams. Also, download the BYJU’S App for interactive study videos.

CBSE Related Links

Leave a Comment Cancel reply

Your Mobile number and Email id will not be published. Required fields are marked *

Request OTP on Voice Call

Post My Comment

case study of agriculture class 10

It was really really helpful

case study of agriculture class 10

Register with BYJU'S & Download Free PDFs

Register with byju's & watch live videos.

Study Rankers

Notes of Ch 4 Agriculture| Class 10th Geography

Study material and notes of ch 4 agriculture class 10th geography, contact form.

case study of agriculture class 10

For the love of farming, the biggest job on Earth

Watch our video

Oilseed Shatter Matters

How a little seam makes the difference in preventing food loss, your phone, your car, your onion, why patents on seeds make sense, goodbye food loss, how the right seed helps growers to overcome a tomato virus, basf agricultural solutions – global website.

Photo of Marko Groydanovic wearing a green hoodie

  • Read the full interview here

Customer Areas

  • 1 Digital Farming
  • 2 Field Crops Seeds & Traits
  • 3 Vegetable Seeds
  • 4 Crop Protection
  • 5 Public Health
  • 6 Professional & Specialty Solutions

Information

  • Author Services

Initiatives

You are accessing a machine-readable page. In order to be human-readable, please install an RSS reader.

All articles published by MDPI are made immediately available worldwide under an open access license. No special permission is required to reuse all or part of the article published by MDPI, including figures and tables. For articles published under an open access Creative Common CC BY license, any part of the article may be reused without permission provided that the original article is clearly cited. For more information, please refer to https://www.mdpi.com/openaccess .

Feature papers represent the most advanced research with significant potential for high impact in the field. A Feature Paper should be a substantial original Article that involves several techniques or approaches, provides an outlook for future research directions and describes possible research applications.

Feature papers are submitted upon individual invitation or recommendation by the scientific editors and must receive positive feedback from the reviewers.

Editor’s Choice articles are based on recommendations by the scientific editors of MDPI journals from around the world. Editors select a small number of articles recently published in the journal that they believe will be particularly interesting to readers, or important in the respective research area. The aim is to provide a snapshot of some of the most exciting work published in the various research areas of the journal.

Original Submission Date Received: .

  • Active Journals
  • Find a Journal
  • Proceedings Series
  • For Authors
  • For Reviewers
  • For Editors
  • For Librarians
  • For Publishers
  • For Societies
  • For Conference Organizers
  • Open Access Policy
  • Institutional Open Access Program
  • Special Issues Guidelines
  • Editorial Process
  • Research and Publication Ethics
  • Article Processing Charges
  • Testimonials
  • Preprints.org
  • SciProfiles
  • Encyclopedia

agriculture-logo

Article Menu

case study of agriculture class 10

  • Subscribe SciFeed
  • Recommended Articles
  • Google Scholar
  • on Google Scholar
  • Table of Contents

Find support for a specific problem in the support section of our website.

Please let us know what you think of our products and services.

Visit our dedicated information section to learn more about MDPI.

JSmol Viewer

Oilseed rape yield prediction from uavs using vegetation index and machine learning: a case study in east china.

case study of agriculture class 10

1. Introduction

2. materials and methods, 2.1. study area and experimental design, 2.2. data acquisition, 2.3. data reduction, 2.4. data modeling, 2.5. model performance.

Click here to enlarge figure

3.1. Oilseed Rape Yield

3.2. reflectance of oilseed rape plants, 3.3. vis of oilseed rape plants, 3.4. pca of vis, 3.5. performance of oilseed rape yield prediction model.

StageModelTraining PerformanceTesting Performance
RMSE (kg/ha)RRMSE (%)R RMSE (kg/ha)RRMSE (%)R
S1RF214.97.570.893531.319.340.243
MLR370.713.050.618559.620.370.297
SVM398.214.020.568518.518.870.314
S2RF167.85.910.925319.711.640.723
MLR309.710.900.733309.511.260.732
SVM316.911.150.722318.211.580.714
S3RF227.78.020.861601.321.880.220
MLR398.614.030.558546.619.890.268
SVM404.0.14.220.558577.021.000.268
S4RF247.48.710.842543.019.760.227
MLR503.817.740.293477.017.360.367
SVM529.018.620.293525.419.120.367

3.6. Comparisons of Predicted and Measured Yield

4. discussion, 5. conclusions, author contributions, institutional review board statement, data availability statement, acknowledgments, conflicts of interest.

  • Clark, R.; Dahlhaus, P.; Robinson, N.; Larkins, J.-A.; Morse-McNabb, E. Matching the model to the available data to predict wheat, barley, or canola yield: A review of recently published models and data. Agric. Sys. 2023 , 211 , 103749. [ Google Scholar ] [ CrossRef ]
  • Habibi, L.N.; Matsui, T.; Tanaka, T.S.T. Critical evaluation of the effects of a cross-validation strategy and machine learning optimization on the prediction accuracy and transferability of a soybean yield prediction model using UAV-based remote sensing. J. Agric. Food Res. 2024 , 16 , 101096. [ Google Scholar ] [ CrossRef ]
  • Luo, L.; Sun, S.; Xue, J.; Gao, Z.; Zhao, J.; Yin, Y.; Gao, F.; Luan, X. Crop yield estimation based on assimilation of crop models and remote sensing data: A systematic evaluation. Agric. Sys. 2023 , 210 , 103711. [ Google Scholar ] [ CrossRef ]
  • Pantazi, X.E.; Moshou, D.; Alexandridis, T.; Whetton, R.L.; Mouazen, A.M. Wheat yield prediction using machine learning and advanced sensing techniques. Comput. Electron. Agric. 2016 , 121 , 57–65. [ Google Scholar ] [ CrossRef ]
  • Ren, S.; Chen, H.; Hou, J.; Zhao, P.; Dong, Q.G.; Feng, H. Based on historical weather data to predict summer field-scale maize yield: Assimilation of remote sensing data to WOFOST model by ensemble Kalman filter algorithm. Comput. Electron. Agric. 2024 , 219 , 108822. [ Google Scholar ] [ CrossRef ]
  • Cheema, M.J.M.; Bakhsh, A.; Mahmood, T.; Liaqat, M.U. Assessment of water allocations using remote sensing and GIS modeling for Indus Basin, Pakistan. In Proceedings of the PSSP Working Paper, Washington, DC, USA, 19 February 2016; pp. 1–42. [ Google Scholar ]
  • Rhebergen, T.; Yeates, S.J. Climate and soil-based constraints to rainfed cotton yield in the Northern Territory, Australia—A modelling approach using APSIM-OZCOT. Eur. J. Agron. 2023 , 151 , 126998. [ Google Scholar ] [ CrossRef ]
  • Nakano, H.; Tanaka, R.; Guan, S.; Ohdan, H. Predicting rice grain yield using normalized difference vegetation index from UAV and GreenSeeker. Crop Environ. 2023 , 2 , 59–65. [ Google Scholar ] [ CrossRef ]
  • Xiong, X.; Zhong, R.; Tian, Q.; Huang, J.; Zhu, L.; Yang, Y.; Lin, T. Daily DeepCropNet: A hierarchical deep learning approach with daily time series of vegetation indices and climatic variables for corn yield estimation. ISPRS J. Photogramm. Remote Sens. 2024 , 209 , 249–264. [ Google Scholar ] [ CrossRef ]
  • Fernandes, J.L.; Ebecken, N.F.F.; Esquerdo, J.C.D.M. Sugarcane yield prediction in Brazil using NDVI time series and neural networks ensemble. Int. J. Remote Sens. 2017 , 38 , 4631–4644. [ Google Scholar ] [ CrossRef ]
  • Sulik, J.J.; Long, D.S. Spectral considerations for modeling yield of canola. Remote Sens. Environ. 2016 , 184 , 161–174. [ Google Scholar ] [ CrossRef ]
  • Wang, Z.; Chen, J.; Zhang, J.; Fan, Y.; Cheng, Y.; Wang, B.; Wu, X.; Tan, X.; Tan, T.; Li, S.; et al. Predicting grain yield and protein content using canopy reflectance in maize grown under different water and nitrogen levels. Field Crops Res. 2021 , 260 , 107988. [ Google Scholar ] [ CrossRef ]
  • Panek, E.; Gozdowski, D. Analysis of relationship between cereal yield and NDVI for selected regions of Central Europe based on MODIS satellite data. Remote Sens. Appl. Soc. Environ. 2020 , 17 , 100286. [ Google Scholar ] [ CrossRef ]
  • Chlingaryan, A.; Sukkarieh, S.; Whelan, B. Machine learning approaches for crop yield prediction and nitrogen status estimation in precision agriculture: A review. Comput. Electron. Agric. 2018 , 151 , 61–69. [ Google Scholar ] [ CrossRef ]
  • Kayad, A.; Rodrigues, F.A., Jr.; Naranjo, S.; Sozzi, M.; Pirotti, F.; Marinello, F.; Schulthess, U.; Defourny, P.; Gerard, B.; Weiss, M. Radiative transfer model inversion using high-resolution hyperspectral airborne imagery—Retrieving maize LAI to access biomass and grain yield. Field Crops Res. 2022 , 282 , 108449. [ Google Scholar ] [ CrossRef ] [ PubMed ]
  • Kaul, M.; Hill, R.L.; Walthall, C. Artificial neural networks for corn and soybean yield prediction. Agric. Sys. 2005 , 85 , 1–18. [ Google Scholar ] [ CrossRef ]
  • Kamir, E.; Waldner, F.; Hochman, Z. Estimating wheat yields in Australia using climate records, satellite image time series and machine learning methods. ISPRS J. Photogramm. Remote Sens. 2020 , 160 , 124–135. [ Google Scholar ] [ CrossRef ]
  • Marques Ramos, A.P.; Prado Osco, L.; Elis Garcia Furuya, D.; Nunes Gonçalves, W.; Cordeiro Santana, D.; Pereira Ribeiro Teodoro, L.; Antonio da Silva Junior, C.; Fernando Capristo-Silva, G.; Li, J.; Henrique Rojo Baio, F.; et al. A random forest ranking approach to predict yield in maize with uav-based vegetation spectral indices. Comput. Electron. Agric. 2020 , 178 , 105791. [ Google Scholar ] [ CrossRef ]
  • Jeevaganesh, R.; Harish, D.; Priya, B. A machine learning-based approach for crop yield prediction and fertilizer recommendation. In Proceedings of the 6th International Conference on Trends in Electronics and Informatics (ICOEI), Tirunelveli, India, 28–30 April 2022; pp. 1330–1334. [ Google Scholar ]
  • Rajković, D.; Marjanović Jeromela, A.; Pezo, L.; Lončar, B.; Zanetti, F.; Monti, A.; Kondić Špika, A. Yield and quality prediction of winter rapeseed—Artificial neural network and random forest models. Agronomy 2021 , 12 , 58. [ Google Scholar ] [ CrossRef ]
  • Liu, Z.; Ju, H.; Ma, Q.; Sun, C.; Lv, Y.; Liu, K.; Wu, T.; Cheng, M. Rice yield estimation using multi-temporal remote sensing data and machine learning: A case study of Jiangsu, China. Agriculture 2024 , 14 , 14040638. [ Google Scholar ] [ CrossRef ]
  • Gómez, D.; Salvador, P.; Sanz, J.; Casanova, J.L. Potato yield prediction using machine learning techniques and sentinel 2 Data. Remote Sens. 2019 , 11 , 1745. [ Google Scholar ] [ CrossRef ]
  • Sun, C.; Feng, L.; Zhang, Z.; Ma, Y.; Crosby, T.; Naber, M.; Wang, Y. Prediction of end-of-season tuber yield and tuber set in potatoes using in-season UAV-based hyperspectral imagery and machine learning. Sensors 2020 , 20 , 5293. [ Google Scholar ] [ CrossRef ] [ PubMed ]
  • Johnson, M.D.; Hsieh, W.W.; Cannon, A.J.; Davidson, A.; Bédard, F. Crop yield forecasting on the Canadian Prairies by remotely sensed vegetation indices and machine learning methods. Agric. For. Meteorol. 2016 , 218–219 , 74–84. [ Google Scholar ] [ CrossRef ]
  • Prasad, N.R.; Patel, N.R.; Danodia, A. Crop yield prediction in cotton for regional level using random forest approach. Spat. Inf. Res. 2020 , 29 , 195–206. [ Google Scholar ] [ CrossRef ]
  • Matese, A.; Toscano, P.; Di Gennaro, S.; Genesio, L.; Vaccari, F.; Primicerio, J.; Belli, C.; Zaldei, A.; Bianconi, R.; Gioli, B. Intercomparison of UAV, aircraft and satellite remote sensing platforms for precision viticulture. Remote Sens. 2015 , 7 , 2971–2990. [ Google Scholar ] [ CrossRef ]
  • Taşan, S.; Cemek, B.; Taşan, M.; Cantürk, A. Estimation of eggplant yield with machine learning methods using spectral vegetation indices. Comput. Electron. Agric. 2022 , 202 , 107367. [ Google Scholar ] [ CrossRef ]
  • Rouse, J.W.; Haas, R.H.; Schell, J.A.; Deering, D.W. Monitoring vegetation systems in the Great Plains with ETRS. In Proceedings of the Third Earth Resources Technology Satellite-1 Symposium, Washington, DC, USA, 10–14 December 1973; pp. 309–317. [ Google Scholar ]
  • Sulik, J.J.; Long, D.S. Spectral indices for yellow canola flowers. Int. J. Remote Sens. 2015 , 36 , 2751–2765. [ Google Scholar ] [ CrossRef ]
  • Gitelson, A.A.; Viña, A.; Ciganda, V.; Rundquist, D.C.; Arkebauer, T.J. Remote estimation of canopy chlorophyll content in crops. Geophys. Res. Lett. 2005 , 32 , L08403. [ Google Scholar ] [ CrossRef ]
  • Jordan, C.F. Derivation of Leaf-Area Index from Quality of Light on the Forest Floor. Ecology 1969 , 50 , 663–666. [ Google Scholar ] [ CrossRef ]
  • Broge, N.H.; Leblanc, E. Comparing prediction power and stability of broadband and hyperspectral vegetation indices for estimation of green leaf area index and canopy chlorophyll density. Remote Sens. Environ. 2001 , 76 , 156–172. [ Google Scholar ] [ CrossRef ]
  • Hancock, D.W.; Dougherty, C.T. Relationships between blue-and red-based vegetation indices and leaf area and yield of alfalfa. Crop Sci. 2007 , 47 , 2547–2556. [ Google Scholar ] [ CrossRef ]
  • Hadjimitsis, D.G.; Papadavid, G.; Agapiou, A.; Themistocleous, K.; Hadjimitsis, M.G.; Retalis, A.; Michaelides, S.; Chrysoulakis, N.; Toulios, L.; Clayton, C.R.I. Atmospheric correction for satellite remotely sensed data intended for agricultural applications: Impact on vegetation indices. Nat. Hazards Earth Syst. Sci. 2010 , 10 , 89–95. [ Google Scholar ] [ CrossRef ]
  • Haboudane, D.; Miller, J.R.; Tremblay, N.; Zarco-Tejada, P.J.; Dextraze, L. Integrated narrow-band vegetation indices for prediction of crop chlorophyll content for application to precision agriculture. Remote Sens. Environ. 2002 , 81 , 416–426. [ Google Scholar ] [ CrossRef ]
  • Alabi, T.R.; Abebe, A.T.; Chigeza, G.; Fowobaje, K.R. Estimation of soybean grain yield from multispectral high-resolution UAV data with machine learning models in West Africa. Remote Sens. Appl. Soc. Environ. 2022 , 27 , 100782. [ Google Scholar ] [ CrossRef ]
  • Ge, H.; Ma, F.; Li, Z.; Du, C. Estimating rice yield by assimilating UAV-derived plant nitrogen concentration into the DSSAT model: Evaluation at different assimilation time windows. Field Crops Res. 2022 , 288 , 108705. [ Google Scholar ] [ CrossRef ]
  • Barriguinha, A.; Jardim, B.; de Castro Neto, M.; Gil, A. Using NDVI, climate data and machine learning to estimate yield in the Douro wine region. Int. J. Appl. Earth Obs. Geoinf. 2022 , 114 , 103069. [ Google Scholar ] [ CrossRef ]
  • Zsebő, S.; Bede, L.; Kukorelli, G.; Kulmány, I.M.; Milics, G.; Stencinger, D.; Teschner, G.; Varga, Z.; Vona, V.; Kovács, A.J. Yield Prediction Using NDVI Values from GreenSeeker and MicaSense Cameras at Different Stages of Winter Wheat Phenology. Drones 2024 , 8 , 88. [ Google Scholar ] [ CrossRef ]
  • Lukas, V.; Huňady, I.; Kintl, A.; Mezera, J.; Hammerschmiedt, T.; Sobotková, J.; Brtnický, M.; Elbl, J. Using UAV to Identify the optimal vegetation index for yield prediction ofoil seed rape ( Brassica napus L.) at the flowering stage. Remote Sens. 2022 , 14 , 4953. [ Google Scholar ] [ CrossRef ]
  • Huang, J.; Tian, L.; Liang, S.; Ma, H.; Becker-Reshef, I.; Huang, Y.; Su, W.; Zhang, X.; Zhu, D.; Wu, W. Improving winter wheat yield estimation by assimilation of the leaf area index from Landsat TM and MODIS data into the WOFOST model. Agric. For. Meteorol. 2015 , 204 , 106–121. [ Google Scholar ] [ CrossRef ]
  • Sadenova, M.; Beisekenov, N.; Varbanov, P.S.; Pan, T. Application of machine learning and neural networks to predict the yield of cereals, legumes, oilseeds and forage crops in Kazakhstan. Agriculture 2023 , 13 , 1195. [ Google Scholar ] [ CrossRef ]
  • Xie, Y.; Wang, P.; Bai, X.; Khan, J.; Zhang, S.; Li, L.; Wang, L. Assimilation of the leaf area index and vegetation temperature condition index for winter wheat yield estimation using Landsat imagery and the CERES-Wheat model. Agric. For. Meteorol. 2017 , 246 , 194–206. [ Google Scholar ] [ CrossRef ]
  • Zhou, H.; Yang, J.; Lou, W.; Sheng, L.; Li, D.; Hu, H. Improving grain yield prediction through fusion of multi-temporal spectral features and agronomic trait parameters derived from UAV imagery. Front. Plant Sci. 2023 , 14 , 1217448. [ Google Scholar ] [ CrossRef ] [ PubMed ]
  • Adnan, R.M.; Liang, Z.; Heddam, S.; Zounemat-Kermani, M.; Kisi, O.; Li, B. Least square support vector machine and multivariate adaptive regression splines for streamflow prediction in mountainous basin using hydro-meteorological data as inputs. J. Hydrol. 2020 , 586 , 124371. [ Google Scholar ] [ CrossRef ]
  • Freitas, R.G.; Pereira, F.R.S.; Dos Reis, A.A.; Magalhães, P.S.G.; Figueiredo, G.K.D.A.; do Amaral, L.R. Estimating pasture aboveground biomass under an integrated crop-livestock system based on spectral and texture measures derived from UAV images. Comput. Electron. Agric. 2022 , 198 , 107122. [ Google Scholar ] [ CrossRef ]
  • He, D.; Zamora, M.; Oto, A.; Karczmar, G.S.; Fan, X. Comparison of region-of-interest-averaged and pixel-averaged analysis of DCE-MRI data based on simulations and pre-clinical experiments. Phys. Med. Biol. 2017 , 62 , N445–N459. [ Google Scholar ] [ CrossRef ] [ PubMed ]
NotationTreatmentNote
F1N fertilizer 125 kg/haRepeated 12 times
F2N fertilizer 500 kg/ha
F3N fertilizer 750 kg/ha
NHBHerbicide (S)-Metolachlor sprayed with 1.5 L/haRepeated 12 times
HBWithout herbicide (S)-Metolachlor sprayed
D1Density of 1.5 × 10 seedlings/haRepeated 6 times
D2Density of 4.5 × 10 seedlings/ha
D3Density of 7.5 × 10 seedlings/ha
CFCompound fertilizer (N:P O :K O = 16:16:16)Repeated 3 times
SFSpecial fertilizer for oilseed rape plant
IndexEquationReference
NDVI(R800 − R670)/(R800 + R670)[ ]
NDVY(R560 − R450)/(R560 + R450)[ ]
CIreR800/R720-1[ ]
CIgreenR800/R550-1[ ]
RVIR800/R670[ ]
TVI0.5 × (120 × (R750 − R550) − 200 × (R670 − R550)[ ]
BNDVI(R860 − R450)/(R860 + R450)[ ]
DVIR800-R670[ ]
RDVI[NDVI × (R800 + R670) ^ 2] ^ 0.5[ ]
NIRR860/
Stage Median (kg/ha)Average (kg/ha)Max (kg/ha)Min (kg/ha)RangeSkewnessKurtosis
Measured2901.32747.73701.91215.02486.8−0.530.20
S1RF2950.92856.93313.81695.91617.9−2.004.11
MLR2928.42744.03304.4643.62660.8−2.235.59
SVM3001.72871.53310.41322.21988.2−2.124.67
S2RF2825.22676.13318.41388.71929.8−2.004.11
MLR2870.22695.43338.11470.11868.0−2.235.59
SVM2885.12718.43279.01591.51687.5−2.124.67
S3RF2856.42704.83603.41477.42125.9−2.004.11
MLR2779.72703.33822.51658.92163.6−2.235.59
SVM2792.82705.83980.71516.02464.8−2.124.67
S4RF2877.92761.63384.01458.51925.5−2.004.11
MLR2842.72825.13401.82182.31219.5−2.235.59
SVM2907.42897.83213.32546.2667.1−2.124.67
The statements, opinions and data contained in all publications are solely those of the individual author(s) and contributor(s) and not of MDPI and/or the editor(s). MDPI and/or the editor(s) disclaim responsibility for any injury to people or property resulting from any ideas, methods, instructions or products referred to in the content.

Share and Cite

Hu, H.; Ren, Y.; Zhou, H.; Lou, W.; Hao, P.; Lin, B.; Zhang, G.; Gu, Q.; Hua, S. Oilseed Rape Yield Prediction from UAVs Using Vegetation Index and Machine Learning: A Case Study in East China. Agriculture 2024 , 14 , 1317. https://doi.org/10.3390/agriculture14081317

Hu H, Ren Y, Zhou H, Lou W, Hao P, Lin B, Zhang G, Gu Q, Hua S. Oilseed Rape Yield Prediction from UAVs Using Vegetation Index and Machine Learning: A Case Study in East China. Agriculture . 2024; 14(8):1317. https://doi.org/10.3390/agriculture14081317

Hu, Hao, Yun Ren, Hongkui Zhou, Weidong Lou, Pengfei Hao, Baogang Lin, Guangzhi Zhang, Qing Gu, and Shuijin Hua. 2024. "Oilseed Rape Yield Prediction from UAVs Using Vegetation Index and Machine Learning: A Case Study in East China" Agriculture 14, no. 8: 1317. https://doi.org/10.3390/agriculture14081317

Article Metrics

Article access statistics, further information, mdpi initiatives, follow mdpi.

MDPI

Subscribe to receive issue release notifications and newsletters from MDPI journals

Long-term sustainability of the water-agriculture-energy nexus in Brazil’s MATOPIBA region: A case study using system dynamics

  • Trajectories of Social-Ecological Systems in the Global South
  • Published: 08 August 2024

Cite this article

case study of agriculture class 10

  • Minella Alves Martins   ORCID: orcid.org/0000-0002-6856-692X 1 ,
  • David Collste 2 ,
  • Francisco Gilney Silva Bezerra 1 ,
  • Marcela Aparecida Campos Neves Miranda 1 ,
  • André Rodrigues Gonçalves 1 ,
  • Jocilene Dantas Barros 1 ,
  • Manoel Ferreira Cardoso 1 ,
  • Amanda Sousa Silvino 1 ,
  • Taís Sonetti-González 3 ,
  • Jean Pierre Henry Balbaud Ometto 1 ,
  • Celso von Randow 1 ,
  • Javier Tomasella 1 &
  • Ana Paula Dutra de Aguiar 1 , 2  

The global demand for agricultural commodities has driven extensive land conversion to agriculture in Brazil, especially in the MATOPIBA region. This area encompasses the Rio Grande Basin, a major tributary of the São Francisco Basin that is known for expanding intensive irrigated agriculture and hydropower generation. However, recent data reveal declining precipitation and aquifer recharge, potentially exacerbating ongoing water and land conflicts. This study investigates the long-term sustainability of agricultural expansion amid the worsening water scarcity using a system dynamics model. Findings suggest that rising costs and decreasing profits due to irrigation water shortages may hinder the expansion of irrigated land. By 2040, the irrigation demand may remain partly unmet, while downstream flow and baseflow could decrease. Additionally, agricultural expansion will significantly raise energy demand, posing a developmental challenge. We suggest that ensuring the sustainability of the Rio Grande Basin depends on improved water management and exploring alternative energy sources to address existing constraints.

This is a preview of subscription content, log in via an institution to check access.

Access this article

Subscribe and save.

  • Get 10 units per month
  • Download Article/Chapter or eBook
  • 1 Unit = 1 Article or 1 Chapter
  • Cancel anytime

Price includes VAT (Russian Federation)

Instant access to the full article PDF.

Rent this article via DeepDyve

Institutional subscriptions

case study of agriculture class 10

Alvares, C.A., J.L. Stape, P.C. Sentelhas, J.L.M. de Gonçalves, and G. Sparovek. 2013. Köppen’s climate classification map for Brazil. Meteorologische Zeitschrift. 22: 711–728. https://doi.org/10.1127/0941-2948/2013/0507 .

Article   Google Scholar  

ANA-National Water Agency. 2021. Water Usage by Municipality. Retrieved 17 March 2023, from https://dadosabertos.ana.gov.br/datasets/1bab1b61623d4594866d47582ccd7181_9/about (In Portuguese).

ANA-National Water Agency. 2021. Atlas of Irrigation–Water Use in Irrigated Agriculture. 2nd Ed. Retrieved 25 November 2022, from http://atlasirrigacao.ana.gov.br/ (In Portuguese).

ANEEL‐National Electric Energy Agency. 2024. Relação de empreendimentos de Geração Distribuída. Retrieved 06 May 2024, from https://dadosabertos.aneel.gov.br/dataset/relacao-de-empreendimentos-de-geracao-distribuida (In Portuguese).

Aragão, A., and E. Contini. 2021. Agriculture in Brazil and the World: A Summary of the Period from 2000 to 2020. EMBRAPA SIRE. Retrieved 10 October 2023, from https://www.embrapa.br/documents/10180/62618376/O+AGRO+NO+BRASIL+E+NO+MUNDO.pdf (In Portuguese).

Barlas, Y. 1996. Formal aspects of model validity and validation in system dynamics. System Dynamics Review 12: 183–210.

Binder, T., A. Vox., S. Belyazid, H. Haraldsson and M. Svensson. 2004. Developing system dynamics models from causal loop diagrams. In Proceedings of the 22nd International Conference of the System Dynamics Society . Oxford. Retrieved 12 May 2023, from https://citeseerx.ist.psu.edu/document?repid=rep1&type=pdf&doi=cf00b9084b05ba357bf0c5fa7a5b9cc1b5695015 .

Calmon, D. 2020. Shifting frontiers: The making of Matopiba in Brazil and global redirected land use and control change. The Journal of Peasant Studies . https://doi.org/10.1080/03066150.2020.1824183 .

CBHSF-São Francisco River Basin Committee. 2021. The construction of a new Small Hydroelectric Plant (PCH) on the Rio Grande raises concerns among residents and traditional communities. Retrieved 04 November 2023, from https://cbhsaofrancisco.org.br/noticias/novidades/construcao-de-nova-pch-no-rio-grande-causa-preocupacao-em-moradores-e-comunidades-tradicionais/ (In Portuguese).

CEPEA/CNA-Center for Advanced Studies on Applied Economics–CEPEA, Brazilian Confederation of Agriculture and Livestock–CAN. 2021. Gross Domestic Product (GDP) of the Brazilian agribusiness from 1996 to 2021. Retrieved 13 March 2023, from https://ww.cepea.esalq.usp.br/br/pib-do-agronegocio-brasileiro.aspx (In Portugues).

Collste, D., M. Pedercini, and S.E. Cornell. 2017. Policy coherence to achieve the SDGs: Using integrated simulation models to assess effective policies. Sustainability Science 12: 921–931. https://doi.org/10.1007/s11625-017-0457-x .

Collste, D., A.P. Aguiar, Z. Harmáčková, D. Galafassi, L. Pereira, O. Selomane, and S. van der Leeuw. 2023. Participatory pathways to the Sustainable Development Goals: Inviting divergent perspectives through a cross-scale systems approach. Environmental Research Communications . https://doi.org/10.1088/2515-7620/acce25 .

CPRM-Mineral Resources Research Company. 2022. SIAGAS: Groundwater Information System. Retrieved 30 November 2022, from http://siagasweb.cprm.gov.br/layout/ (In Portuguese).

CPT-Pastoral Land Commission. 2022. Land and Water Conflits. Retrieved 15 March 2023, from https://www.cptnacional.org.br/ (In Portugues).

Dionizio, E.A., and M.H. Costa. 2019. Influence of land use and land cover on hydraulic and physical soil properties at the Cerrado agricultural frontier. Agriculture 9: 24. https://doi.org/10.3390/agriculture9010024 .

Dionizio, E.A., F.M. Pimenta, L.B. Lima, and M.H. Costa. 2020. Carbon stocks and dynamics of different land uses on the Cerrado agricultural frontier. PLoS ONE 15: e0241637. https://doi.org/10.1371/journal.pone.0241637 .

Article   CAS   Google Scholar  

Eger, G.Z.S., G.C. Silva Junior, E.A.G. Marques, B.R.C. Leão, D.G.T.B. da Rocha, T.E. Gilmore, L.G.H. Amaral, J.A.O. Silva, et al. 2021. Recharge assessment in the context of expanding agricultural activity: Urucuia Aquifer System, western State of Bahia Brazil. Journal of South American Earth Sciences. https://doi.org/10.1016/j.jsames.2021.103601 .

EMBRAPA-Brazilian Agricultural Research Corporation. 2018. VISION 2030: The Future of Brazilian Agriculture. Retrieved 13 March 2023, from https://www.embrapa.br/visao/trajetoria-da-agricultura-brasileira (In Portugues).

EPE-Energy Research Company. Balanço Energético Nacional. 2023. Retrieved 06 May 2024, from https://www.epe.gov.br/pt/publicacoes-dados-abertos/publicacoes/balanco-energetico-nacional-2023 (In Portuguese).

Fernald, A.G., S.Y. Cevik, C.G. Ochoa, V.C. Tidwell, J.P. King, and S.J. Guldan. 2010. River hydrograph retransmission functions of irrigated valley surface water-groundwater interactions. Journal of Irrigation and Drainage Engineering. 136: 823–835. https://doi.org/10.1061/(ASCE)IR.1943-4774.0000265 .

Forrester, J.W. 1969. Urban Dynamics . Cambridge: The M.I.T. Press.

Google Scholar  

Gaspar, M. T. P., and J.E.G. Campos. 2007. The Urucuia Aquifer System. Revista Brasileira de Geociências 37. Sociedade Brasileira de Geologia: 216–226. https://doi.org/10.25249/0375-7536.200737s4216226 . (In Portuguese).

Gonçalves, R.D., B.Z. Engelbrecht, and H.K. Chang. 2016. Hydrological analysis of historical data in the Rio Grande Basin (BA): Contribution of the Urucuia Aquifer System. Águas Subterrâneas 30: 190–208. https://doi.org/10.14295/ras.v30i2.28514 . (In Portuguese).

Gonçalves, R. D. and H. K. Chang. 2017. Hydrogeological model of the Urucuia Aquifer System in the Rio Grande Basin (BA) Geociências, 36:205–220. Retrieved 11 October 2023, from https://www.periodicos.rc.biblioteca.unesp.br/index.php/geociencias/article/view/11132/8152 . (In Portuguese).

Harms, J.Z., J.J. Malard-Adam, J.F. Adamowski, A. Sharma, and A. Nkwasa. 2023. Dynamically coupling system dynamics and SWAT+ models using Tinamït: application of modular tools for coupled human–water system models. Hydrology and Earth System Sciences 27: 1683–1693. https://doi.org/10.5194/hess-27-1683-2023 .

Herraiz, J.I., R.H. Almeida, M. Castillo-Cagigal, and L. Narvarte. 2023. Experimental performance evaluation of a PV-powered center-pivot irrigation system for a three-year operation period. Energies 16: 3654. https://doi.org/10.3390/en16093654 .

Hofmann, G.S., R.C. Silva, E.J. Weber, A.A. Barbosa, L.F.B. Oliveira, R.J.V. Alves, H. Hasenack, V. Schossler, et al. 2023. Changes in atmospheric circulation and evapotranspiration are reducing rainfall in the Brazilian Cerrado. Scientific Reports 13: 11236. https://doi.org/10.1038/s41598-023-38174-x .

IBGE-Brazilian Institute of Geography and Statistics. 2022. Brazilian Census of 2022. Rio de Janeiro: IBGE. Retrieved 10 September 2023, from https://censo2022.ibge.gov.br/ (In Portuguese).

INEMA-Institute for the Environment and Water Resources. 2022. Normative Instruction nº 003 of June 30, 2022. Retrieved 08 July 2024, from http://www.inema.ba.gov.br/wp-content/files/INEMA_-_Instrucao_Normativa_003_-_Procedimentos_Administrativos_e_Critrios_de_Autorizao_para_Perfurao_de_Poos_-_APPO__no_aqufero_Urucuia.pdf . (In Portuguese).

Jeong, H., and J. Adamowski. 2016. A system dynamics based socio-hydrological model for agricultural wastewater reuse at the watershed scale. Agricultural Water Management 171: 89–107. https://doi.org/10.1016/j.agwat.2016.03.019 .

Lopes, G.R., M.G.B. Lima, and T.N.P. dos. Reis. 2021. Maldevelopment revisited: Inclusiveness and social impacts of soy expansion over Brazil’s Cerrado in Matopiba. World Development 139: 105316. https://doi.org/10.1016/j.worlddev.2020.1053 .

Luiz, E.W., F.R. Martins, A.R. Gonçalves, and E.B. Pereira. 2018. Analysis of intra-day solar irradiance variability in different Brazilian climate zones. Solar Energy 167: 210–219. https://doi.org/10.1016/j.solener.2018.04.005 .

Marengo, J.A., J.C. Jimenez, J.C. Espinoza, A.P. Cunha, and L.E.O. Aragão. 2022. Increased climate pressure on the agricultural frontier in the Eastern Amazonia-Cerrado transition zone. Scientific Reports 12: 457. https://doi.org/10.1038/s41598-021-04241-4 .

Marques, E.A.G., G.C. Silva Junior, G.Z.S. Eger, M.A. Ilambwetsi, R. Pousa, T.N. Generoso, J. Oliveira, and J.N. Júnior. 2020. Analysis of groundwater and river stage fluctuations and their relationship with water use and climate variation effects on Alto Grande watershed, Northeastern Brazil. Journal of South American Earth Sciences 103: 102723. https://doi.org/10.1016/j.jsames.2020.102723 .

Martins, M.A., J. Tomasella, and C.G. Dias. 2019. Maize yield under a changing climate in the Brazilian Northeast: Impacts and adaptation. Agricultural Water Management 216: 339–350. https://doi.org/10.1016/j.agwat.2019.02.011 .

Meadows, D. 2008. Thinking in Systems: A Primer . White River Junction, Vermont: Chelsea Green Publishing.

Moallemi, E.A., S.H. Hosseini, S. Eker, L. Gao, E. Bertone, K. Szetey, and B.A. Bryan. 2022. Eight archetypes of sustainable development goal (SDG) synergies and trade-offs. Earth’s Future 10: e2022EF002873. https://doi.org/10.1029/2022EF002873 .

Multsch, S., M.S. Krol, M. Pahlow, A.L.C. Assunção, A.G.O.P. Barretto, Q.J. de van Lier, and L. Breuer. 2020. Assessment of potential implications of agricultural irrigation policy on surface water scarcity in Brazil. Hydrology and Earth System Sciences. 24: 307–324. https://doi.org/10.5194/hess-24-307-2020 .

Pereira, E. B., F.R. Martins, A.R. Gonçalves, R.S. Costa, F.L. Lima, R. Rüther, S.L. Abreu, G.M. Tiepolo, S.V. Pereira, and J.G. Souza. 2017. Brazilian Atlas of Solar Energy. 2.ed. São José dos Campos: INPE, 2017. 80p. Retrieved 20 September 2023, from https://doi.org/10.34024/978851700089 (In Portuguese).

Pereira, B.H.F., C. Dereczynski, G.C. da Silva Junior, and E.A.G. Marques. 2022. Projected climate change impacts on groundwater recharge in the Urucuia aquifer system Brazil. International Journal of Climatology 42: 8822–8838. https://doi.org/10.1002/joc.7773 .

Phan, T.D., E. Bertone, and R.A. Stewart. 2021. Critical review of system dynamics modelling applications for water resources planning and management. Cleaner Environmental Systems 2: 100031. https://doi.org/10.1016/j.cesys.2021.100031 .

Pimenta, F.M., A.T. Speroto, M.H. Costa, and E.A. Dionizio. 2021. Historical Changes in Land Use and Suitability for Future Agriculture Expansion in Western Bahia Brazil. Remote Sensing 13: 1088. https://doi.org/10.3390/rs13061088 .

Pousa, R., M.H. Costa, F.M. Pimenta, V.C. Fontes, V.F.A. de Brito, and M. Castro. 2019. Climate change and intense irrigation growth in Western Bahia, Brazil: The urgent need for hydroclimatic monitoring. Water 11: 933. https://doi.org/10.3390/w11050933 .

Pousa, R., and M.H. Costa. 2023. Interactions between large-scale and mesoscale processes define long-term rainfall variability and availability of water resources in Western Bahia, Brazil. International Journal of Climatology 43: 3416–3432. https://doi.org/10.1002/joc.8036 .

Pignati, W., F. Lima, S. Lara, M. Correa, J. Barbosa, L. Leão, and M. Pignatti. 2017. Spatial distribution of pesticide use in Brazil: a strategy for Health Surveillance. Ciência e Saúde Coletiva 22: 3281–3293. https://doi.org/10.1590/1413-812320172210.17742017 . (In Portugues).

Reis, P. A. G. 2018. Evaluation of flow contributions from the Rio Grande watershed to the São Francisco River channel to meet multiple uses. University of Bahia. Salvador. 163p. Retrieved 10 August 2023, from https://maasa.ufba.br/sites/maasa.ufba.br/files/dissertacao_de_mestrado_-_polyana_alcantara_galvao.pdf . (In Portugues).

Santos, C. A. P. 2016. Environmental Education as a Tool for Conflict Mitigation in the Rio Grande Watershed, Western Bahia. HOLOS. 8: 156–172. Retrieved 5 April 2023, from https://doi.org/10.15628/holos.2016.2837 . (In Portuguese).

Santos, C.A.P., E.E. Sano, and P.S. Santos. 2018. Agricultural frontier and dynamic of land use and ocuppation in western bahia. ACTA Geográfica, Boa Vista. 12: 17–32 (In Portuguese).

Senge, P. M. (1990). The fifth discipline: The art and practice of the learning organization (1. Currency paperback ed). Currency Doubleday.

Simões, Y.D.S., E.H.B.C. Silva, and H.A. de Araújo. 2018. Rainfall zoning of Bahia State, Brazil: An update proposal. Ambiente e Agua - an Interdisciplinary Journal of Applied Science 13: 1. https://doi.org/10.4136/ambi-agua.2171 .

Sousa, M.A., J. Alves Júnior, A.W.P. Evangelista, D. Casaroli, and M. Mesquita. 2019. Estimativa de viabilidade econômica do uso de energia fotovoltaica em pivô central no estado de Goiás. Revista Engenharia Na Agricultura - REVENG 27: 22–29. https://doi.org/10.13083/reveng.v27i1.848 .

Souza Jr, C.M., J.Z. Shimbo, M.R. Rosa, L.L. Parente, A.A. Alencar, B.F.T. Rudorff, H.H.M. Matsumoto, L.G. Ferreira, et al. 2020. Reconstructing three decades of land use and land cover changes in Brazilian biomes with landsat archive and earth engine. Remote Sensing 12: 2735. https://doi.org/10.3390/rs12172735 .

Sterman, J.D. 2000. Business dynamics - systems thinking and modeling for a complex world . Boston: McGraw-Hill.

Sušnik, J., L.S. Vamvakeridou-Lyroudia, D.A. Savić, and Z. Kapelan. 2012. Integrated system dynamics modelling for water scarcity assessment: case study of the Kairouan region. Science of the Total Environment 440: 290–306. https://doi.org/10.1016/j.scitotenv.2012.05.085 .

Todde, G., L. Murgia, P.A. Deligios, R. Hogan, I. Carrelo, M. Moreira, A. Pazzona, L. Ledda, et al. 2019. Energy and environmental performances of hybrid photovoltaic irrigation systems in Mediterranean intensive and super-intensive olive orchards. Science of the Total Environment 651: 2514–2523. https://doi.org/10.1016/j.scitotenv.2018.10.175 .

Valdiones, A. P., P. Bernasconi, V. Silgueiro, V. Guidotti, F. Miranda, J. Costa, R. Rajão, B. Manzolli. 2021. Illegal Deforestation and Conversion in the Amazon and Matopiba: lack of transparency and access to information. Retrieved 04 October 2023, from https://www.icv.org.br/website/wp-content/uploads/2021/05/icv-relatorio-ing-v1-1-1.pdf .

Zomorodian, M., S.H. Lai, M. Homayounfar, S. Ibrahim, S.E. Fatemi, and A. El-Shafie. 2018. The state-of-the-art system dynamics application in integrated water resources modeling. Journal of Environmental Management. 227: 294–304. https://doi.org/10.1016/j.jenvman.2018.08.097 .

Download references

Acknowledgements

The authors would like to thank the NEXUS Project, funded by the São Paulo Research Foundation–FAPESP, under Grant Numbers 2022/00917-0 and 2017/22269-2. DC, FGBS, TSG, and APDA acknowledge support from the XPaths project—Science in Action: Intersecting Pathways to the SDGs Across Scales in Drylands. CvR acknowledges CNPq grant number 314780/2020-3, and JT acknowledges CNPq grant number 304695/2020-3.

Author information

Authors and affiliations.

Impacts, Adaptation and Vulnerability Division, National Institute for Space Research (INPE), Astronautas Avenue, 1758, São José dos Campos, SP, 12227-010, Brazil

Minella Alves Martins, Francisco Gilney Silva Bezerra, Marcela Aparecida Campos Neves Miranda, André Rodrigues Gonçalves, Jocilene Dantas Barros, Manoel Ferreira Cardoso, Amanda Sousa Silvino, Jean Pierre Henry Balbaud Ometto, Celso von Randow, Javier Tomasella & Ana Paula Dutra de Aguiar

Stockholm Resilience Centre, Stockholm University, Albanovägen 28, 106 91, Stockholm, Sweden

David Collste & Ana Paula Dutra de Aguiar

Socio-Environmental Dynamics Research Group, Free University of Brussels, Campus du Solbosch-Av. F.D. Roosevelt, 50D, 1050, Brussels, Belgium

Taís Sonetti-González

You can also search for this author in PubMed   Google Scholar

Corresponding author

Correspondence to Minella Alves Martins .

Ethics declarations

Conflict of interest.

All authors declare that they have no conflicts of interest.

Additional information

Publisher's note.

Springer Nature remains neutral with regard to jurisdictional claims in published maps and institutional affiliations.

Supplementary Information

Below is the link to the electronic supplementary material.

Supplementary file1 (PDF 609 KB)

Rights and permissions.

Springer Nature or its licensor (e.g. a society or other partner) holds exclusive rights to this article under a publishing agreement with the author(s) or other rightsholder(s); author self-archiving of the accepted manuscript version of this article is solely governed by the terms of such publishing agreement and applicable law.

Reprints and permissions

About this article

Martins, M.A., Collste, D., Bezerra, F.G.S. et al. Long-term sustainability of the water-agriculture-energy nexus in Brazil’s MATOPIBA region: A case study using system dynamics. Ambio (2024). https://doi.org/10.1007/s13280-024-02058-9

Download citation

Received : 16 October 2023

Revised : 14 March 2024

Accepted : 10 July 2024

Published : 08 August 2024

DOI : https://doi.org/10.1007/s13280-024-02058-9

Share this article

Anyone you share the following link with will be able to read this content:

Sorry, a shareable link is not currently available for this article.

Provided by the Springer Nature SharedIt content-sharing initiative

  • Agricultural expansion
  • Climate change
  • Irrigation demand
  • Sustainability
  • Water scarcity
  • Water management
  • Find a journal
  • Publish with us
  • Track your research

IMAGES

  1. **Agriculture**Class 10th**PPT**

    case study of agriculture class 10

  2. Agriculture Notes for Class 10 Social Science (PDF)

    case study of agriculture class 10

  3. Agriculture Class 10 Notes PDF Download

    case study of agriculture class 10

  4. **Agriculture**Class 10th**PPT**

    case study of agriculture class 10

  5. Agriculture: Class 10 Geography NCERT Chapter 4

    case study of agriculture class 10

  6. **Agriculture**Class 10th**PPT**

    case study of agriculture class 10

COMMENTS

  1. Case Study Questions Class 10 Social Science Geography

    Important Case Study Questions for Class 10 Board Exam Students. Here we have arranged some Important Case Base Questions for students who are searching for Paragraph Based Questions Agriculture. At Case Study Questions there will given a Paragraph. In where some Important Questions will made on that respective Case Based Study.

  2. Case Study Questions Chapter 4 Agriculture

    Chapter 4 Agriculture Case Study Questions Class 10 Social Science. 1. Read the source given below and answer the following questions: Globalisation is not a new phenomenon. It was there at the time of colonisation. In the nineteenth century when European traders came to India, at that time too, Indian spices were exported to different ...

  3. Case Study Questions for Class 10 Social Science Geography Chapter 4

    Case Study Questions for Class 10 Social Science Geography Chapter 4 Agriculture Here we are providing case study questions for Class 10 Social Science Geography Chapter 4 Agriculture. Case Study Question 1: Read the source given below and answer the questions that follow by choosing the most appropriate option: There has been a gradual shift … Continue reading Case Study Questions for Class ...

  4. Case Study/ Passage Based Questions Chapter 4 Agriculture

    STEP 1: Read the case study and questions carefully. Read the case and associated questions carefully. Highlight the main points of the case and any issues that you can identify. Read the questions closely and analyse what they are requiring you to do. Read the case again, linking the information that is relevant to each question you have been ...

  5. NCERT Solutions For Class 10 Geography Social Science Chapter 4

    The main topics that students will learn in Chapter 4 of NCERT Solutions for Class 10 Geography are: Types of farming. Cropping pattern. Major crops. Food crops other than grains. Non-food crops. Technological and institutional reforms. Contribution of agriculture to the national economy, employment and output.

  6. Social Science Class 10 Important Questions Geography Chapter 4 Agriculture

    2. It is generally done on small land holdings which are economically not viable. 2. It is practiced on large pieces of land on scientific and commercial lines. 3. Primitive tools and animals are used for carrying out agricultural activities. 3. In this type of agriculture, machines and modern technology are used. 4.

  7. Case Based Questions Test: Agriculture

    Case Based Questions Test: Agriculture for Class 10 2024 is part of Class 10 preparation. The Case Based Questions Test: Agriculture questions and answers have been prepared according to the Class 10 exam syllabus.The Case Based Questions Test: Agriculture MCQs are made for Class 10 2024 Exam. Find important definitions, questions, notes, meanings, examples, exercises, MCQs and online tests ...

  8. NCERT Solutions for Class 10 Geography Chapter 4 Agriculture

    Class 10 Geography NCERT Solutions provides detailed answers to all questions in the NCERT textbook. Table of Content. 1. NCERT Solutions for Class 10 Geography Chapter 4 - FREE PDF Download. 2. Glance on NCERT Solutions for Geography Class 10 Chapter 4 Agriculture. 3. Access NCERT Solutions for Class 10 Geography Chapter 4 Agriculture. 4.

  9. NCERT Solutions for Class 10 Geography Chapter 4 Agriculture

    NCERT Solutions for Class 10 Geography Chapter 4 Agriculture: Chapter Overview. India is an agriculturally important country. Two-thirds of its population is engaged in agricultural activities. Agriculture is a primary activity, which produces most of the food that we consume. Besides food grains, it also produces raw material for various ...

  10. Chapter 4: Agriculture

    Solutions of Chapter 4 Agriculture NCERT Social Science Geography Class 10. Meticulously crafted NCERT solutions to boost your CBSE board exam preparation. Easily score more marks in Geography CBSE exam.

  11. CBSE Class 10 Geography Agriculture Case Study Questions

    Agriculture Case Study Questions (CSQ's) Select the number of questions for the test: Keep paper and pencil ready but keep your books away. You can move between questions and answer them in any order you like. These tests are unlimited in nature…take as many as you like. You will be able to view the solutions only after you end the test.

  12. Agriculture Class 10 Notes CBSE Geography Chapter 4 (PDF)

    Agriculture Class 10 Notes Geography Chapter 4 - PDF Download. Class 10 Geography Chapter 4 is Agriculture. Agriculture engages two-third of India's population and is the primary activity which produces raw material for a wide range of industries. In the Agriculture notes, you will study the different types of farming methods, major crops ...

  13. Agriculture Class 10 Important Questions Social Science Geography

    India is the largest producer as well as consumer of pulses in the world. Short Answer Type Questions [3 Marks] Question 10. Mention any three geographical conditions required for rice cultivation in India. Answer: The term used for grouping of small land holdings with bigger ones is consolidation. of land holdings.

  14. Case study questions for CBSE 10th

    Download case study question pdfs for CBSE Class 10th Maths, CBSE Class 10th English, CBSE Class 10th Sciece, CBSE Class 10th SST. As the CBSE 10th Term-1 Board Exams are approaching fast, you can use these worksheets for FREE for practice by students for the new case study formats for CBSE introduced this year.

  15. Question 1

    This is the second most important cereal crop. It is the main food crop, in north and north-western part of the country. This rabi crop requires a cool growing season and a bright sunshine at the time of ripening. It requires 50 to 75 cm of annual rainfall evenly-distributed over the growing season. There are two important wheat-growing zones ...

  16. NCERT Solutions for Class 10th: Ch 4 Agriculture Geography

    2. Answer the following questions in 30 words. (i) Name one important beverage crop and specify the geographical conditions required for its growth. (ii) Name one staple crop of India and the regions where it is produced. (iii) Enlist the various institutional reform programmes introduced by the government in the interest of farmers.

  17. Agriculture Class 10 Geography Chapter 4 Extra Questions ...

    10. Mention any two factors which play an important role in the development of plantations. Answer: (i) Developed network of transport and communication connecting the plantation areas. (ii) Developed market. 11. Name the cropping seasons of India with examples. Answer: (i) Rabi - wheat. (ii) Kharif - paddy.

  18. NCERT Solutions For Class 10 Geography Chapter 4 Agriculture

    Answer-. NCERT Solution for Class 10th Chapter 4: Agriculture gives you knowledge of a variety of topics, and you'll develop your recall and response speed. The strain of the lengthy board test syllabus will be less difficult for students to manage. With the aid of these methods, you might study methodically and outperform your classmates.

  19. CBSE Notes Class 10 Geography Chapter 4

    Agriculture is a primary activity which produces most of the food raw material for various industries. In CBSE Notes Class 10 Geography Chapter 4 - Agriculture, you will study the various types of farming, cropping patterns and major crops grown in India. In the end, you will know how much Agriculture contributes to the national economy ...

  20. Agriculture Class 10 Important Questions with Answers

    3. In this type of agriculture, machines and modern technology are used. 4. In this, modern agricultural inputs, e.g., fertilisers and irrigation are not widely used. 4. There is higher use of modern agricultural in­puts, e.g., HYV seeds, fertilisers, irrigation, etc. are used to obtain higher yields and production. 5.

  21. CBSE Notes Class 10 Geography Chapter 4

    Last Updated : 20 Oct, 2023. CBSE Notes Class 10 Geography Chapter 4- Agriculture is about agriculture and the agricultural sector in India. Agriculture is the prime activity of the economy of India and produces raw materials for various types of industries. We will focus on and study the types of farming, cropping patterns, and also the ...

  22. CBSE Class 10 Geography Chapter 4 Agriculture Important Questions

    One of the best ways to understand these concepts thoroughly is to solve the CBSE Class 10 Geography Chapter 4 Agriculture Important Questions. It will also give students an idea about the types of questions that are likely to be repeated in the board exams. Practising these questions will help in scoring high marks on the Social Science paper.

  23. Notes of Ch 4 Agriculture| Class 10th Geography

    Notes of Ch 4 Agriculture| Class 10th Geography. Introduction. • India is an agriculturally important country as two-thirds of its population is engaged in agricultural activities. Types of Farming. • There are various types of farming systems in different parts of India are: → Primitive Subsistence Farming: It is a 'slash and burn ...

  24. Agriculture

    Global Website about Seeds & Traits and Crop Protection, Digital Farming, Public Health, Urban & Rural Pest Control, Turf & Ornamentals and Animal Nutrition

  25. Agriculture

    Yield prediction is an important agriculture management for crop policy making. In recent years, unmanned aerial vehicles (UAVs) and spectral sensor technology have been widely used in crop production. This study aims to evaluate the ability of UAVs equipped with spectral sensors to predict oilseed rape yield. In an experiment, RGB and hyperspectral images were captured using a UAV at the ...

  26. Long-term sustainability of the water-agriculture-energy nexus in

    Case study area. Located in the southeastern part of MATOPIBA (Fig. 1), the Rio Grande Basin is considered one of the most important of the region due to its abundant water resources.It is a subbasin of Brazil's significant São Francisco River Basin, covering around 76 000 km 2 and overlaying more than a third of the Urucuia Aquifer System—UAS. The UAS is mostly an unconfined groundwater ...

  27. How rigid business approaches paved the way for inclusive business

    Acknowledgments. This work was supported by the International Fund for Agricultural Development (IFAD) within the "Greening livestock: Incentive-Based Interventions for Reducing the Climate Impact of Livestock in East Africa" project [grant numbers 2000000994, 2016] implemented by the International Livestock Research Institute (ILRI) and the Centre for International Forestry Research (CIFOR).